Sei sulla pagina 1di 113

Anatomy revision for MRCOG

Of course one of the most important and major subjects (Paper 1 and we are talking about 25-30
SBAs)
Honestly this subject must be covered very well and I can’t recommend ignoring any topic within it,
but let’s have a general overview: (your main source is Oxford revision notes, get some help from
any available Atlas of anatomy you have)
I- Truncal surface anatomy (vertebral levels and corresponding structures), very important but you
may find some controversies between books (For me I went with what was written in Oxford revision
notes).
II- Muscles of abdominal wall (Anterior and posterior), Rectus sheath, inguinal ligament and canal, femoral
sheath and triangle, Structure of spermatic cord.
III- Ligaments within peritoneal cavity (and their embryological origins).
IV- Pelvis ( I think I don’t need to say anything about it :D of course every single word is important):
a. Bones and diameters.
b. Muscles.
c. Fascia and ligaments.
d. Pelvic floor (Memorize the levator ani muscle by heart).
e. Perineal pouches and perineal body.
f. Ischio-rectal fossa (Very important).
V- Rectum and anal canal + (external anal sphincter).
VI- Blood supply of Colon.
VII- Branches of Celiac trunk.
VIII- Kidney, Bladder and ureter and urethra.
IX- Male and female genital systems.
X- Branches of aorta (and their levels).
XI- Blood vessels of pelvis and anterior abdominal wall.
XII- Lymphatic drainage (internal and external genital organs – other pelvic organs - anterior abdominal wall).
XIII- Peripheral nerves in pelvis.
XIV- Pituitary gland.
XV- Fetal skull.
XVI- Breast (you can read that topic from essential revision guide).
So, those are the main topics :D actually they are nearly all topics in the book:D but again don’t panic, you
have enough time, anatomy is important not only to that exam but to all levels you will pass through and for
sure your practice and don’t forget the general rules while studying.
Any comments or notes from you all and especially from seniors are very welcome. GOOD LUCK

63.False statement among following:


A) Urinary bladder is lined by transitional epithelium
B) Parasympathetic supply via splanchnics nerve is motor to detrusor and inhibitory to sphincter
C) Urethra cotain inner longitudinal smooth muscle layer which shorten during micturition and outer
voluntary striated circular muscle which has a role in urinary incontinace
D) Breast is modified apocrine sweat gland
E) Growth of duct in breast is stimulated by progesterone

Which of the following statements about episiotomy is FALSE:


A. Median (midline) episiotomy is generally considered to be less painful than mediolateral episiotomy
B. Mediolateral or lateral episiotomy may be associated with more blood loss than median one.
C. Indications for episiotomy include avoiding an imminent Perineal tear,
the use of forceps, breech delivery, & the delivery of premature infants.
D. The earlier the episiotomy is done during delivery, generally the more beneficial it will be un speeding
delivery.
E. Episiotomy incisions are repaired anatomically in layers.

An obstetrician performs a mediolateral episiotomy to expand the birth canal during a child birth. Which
of the following muscles is typically incised during this procedure?
A. Bulbospongiosus and superficial transverse perineal muscles
B. Bulbospongiosus and deep transverse perineal muscles
C. Bulbospongiosus and ischiocavernosu muscles
D. Ischiocavernosus and levator ani muscles
E. Bulbospongiosus and levator ani muscles

What three muscles form the superficial group of muscles of the perineum?
A.Bulbospongiosus, superficial transverse perineal muscle, and ischiocavernous
B.Deep transverse perineal muscle, external anal sphincter, and bulbospongiosus
C.External vaginal sphincter, deep transverse perineal muscle, and external anal sphincter
D.Iliococceageus, ischiocavernosus, and bulbospongiosus
E.Levator ani, ischiocavernous, and bulbospongiosus

62.False statement regarding anal canal is


A) It is 4 cm long and external anal sphincter is striated one (extension of voluntary levator ani muscle)
B) External anal sphincter is supplied by pudendal nerve
C) Internal anal sphincter is supplied by inferior hypogastric plexus and nervi erigentes
D) Below pectenline mucosa changes from columnar to squamous
E) Lymphatic drainage below intersphincteric groove is to rectum

61)False statement about rectum is:


A) 15 cm long
B) Acute angulation is produce by puborectalis
C) Has no taeniae or appendices epiploicae
D) Anterosuperiorly pouch of Douglas lies
E) Coloproctologist should be called if anorectal mucosal tear in vaginal delivery remain below
puborectalis

60)Wrong statement among following is:


A) Vagina is lined by keratinized stratified squamous epithelium
B) Labia majora contain hair follicle, sebaceous and apocrine gland
C) Vestibule describe as area in between clitoris anteriorly, labia minora anteriorly, fourchette
posteriorly and hymen superiorly
D) Greater vestibular gland of Bartholin`s open at 5 and 7 o`clock position posterolaterally at vaginal
introitus
E) Nerve supply of vulva is iliohypogastric,ilioinguinal, and pudendal nerve

59.True statement among following:


A) Uterus is maintain in anteverted position by transverse cervical ligament
B) Ureter lies above uterine vessels at level of isthmus
C) Patent processus vaginalis(canal of Nuck) leads to indirect inguinal hernia in female
D) Round ligament made up of smooth muscle fibres
E) Fallopian tube has inner longitudinal smooth muscle layer.

58.False statement regarding boundaries of ovarian fossa.


A) Obturator vessels and nerves lies medially
B) Ampulla of uterine tube on medial surface
C) Bifurcation of common iliac artery above
D) Ureter and internal iliac artery and vein behind
E) Posterior border of ovary lies free of mesovarium

57.Pelvic peritoneum is supplied by:


A) Perineal branch of S4
B) Obturator nerve(L2-4)
C) Pudendal nerve
D) Nerve to obturator internus
E) Sciatic nerve

56.Pudendal nerve block is given by piercing needle just beyond ischial spine. Good test of efficacy is
A) Loss of anal reflex
B) Relaxation of pelvic floor
C) Loss of sensation of vulva
D) Loss of sensation of lower third of vagina
E) All of the above

55.False statement among following about ischiorectal fossa is :


A) Posteriorly, the sacrotuberous ligament and gluteus maximus muscle
B) Medially fascia over levator ani and external anal sphincter
C) Laterally ischial tuberosity with obturator internus muscle
D) Pudendal (alcock) canal medially
E) Posteriorly, perineal branch of S4 and perforating cutaneous branch

54.Which of the following is not contain of superficial perineal pouch


A) Bulbospongiosus muscle
B) Ischiocavernosus muscle
C) Superficial transverse perineal muscle
D) External urethral sphincter
E) Corpus spongiosum

53.Pudendal nerve arises from


A) S2 to S4
B) S1 to S5
C) L4, L5,S1
D) L5 S1-2
E) S4

52.Which of the following is not branch of internal iliac artery:


A) Uterine artery
B) Ovarian artery
C) Inferior gluteal artery
D)Middle rectal artery
E) Internal pudendal artery
51.35 year old patient complain of pain in knee jont. USG 1 month back suggestive of ovarian cyst. What
can be cause of pain in knee joint:
A) Obturator nerve irritation
B) Lateral cutaneous nerve of thigh irritation
C) Femoral nerve irritation
D) Sacral nerve irritation
E) Genitofemoral nerve irritation

50.False statement among following is:


A) Right ovarian artery crosses inferior vena cava and is crossed by terminal ileal vein, caecal vein,
ileocolic vein
B) Right vein drain into right renal vein
C) Left common iliac artery is crossed by superior mesenteric vessels
D) Inferior epigastric artery and circumflex iliac artery arises just above inguinal ligament
E) Genitofemoral nerve lies anterior to psoas muscle

49).Right renal artery passes behind:


A) Diaphragm
B) Inferior vena cava
C) Aorta
D) Psoas muscle
E) Ureter

48.Which of the following is false related to acute appendicitis in preganancy:


A) Signs can be relatively subtle
B) Progression of pathology can be rapid due to failure of omentum to access problem and seal it off
C) Upward displacement of caecum
D) Surgical incision of appendicectomy at mc Burney`s point
E) Non of above

47.True statement among following is:


A) Superior pancreaticoduodenal artery is branch of superior mesenteric artery.
B) Inferior mesenteric artery is a branch of superior mesenteric artery
C) Superior mesenteric artery passes over superior to third part of duodenum
D) Inferior mesenteric artery terminate as middle rectal artery
E) Splenic artery give right gastro epiploic artery

46.Wrong statement regarding epiploic foramen is:


A) Superiorly caudate lobe of liver
B) Inferiorly first part of duodenum
C) Posteriorly inferior vena cava,
D) Anteriorly portal vein, hepatic artery, common bile duct
E) Left adrenal gland posteriorly
45.Wrong statement regarding femoral sheath is :
A) Femoral nerve lies in sheath
B) Femoral artery lies in sheath
C) Femoral vein lies in sheath
D) Femoral canal lies in sheath
E) Lacunar ligament lies medialy

44.Which of the following is not component of spermatic cord :


A) Vas deferens
B) Genital branch of genitofemoral nerve
C) Cremasteric artery
D) Inferior epigastric artery
E) Pampiniform venous plexous

43.Deep inguinal ring which is defect in fascia transversalis, is lies at:


A) Mid-inguinal canal point
B) Mid point of inguinal ligament
C) Divergent fibres of external oblique
D) Heselbach triangle
E) Femoral ring

42.Pyramidalis is supplied by:


A) Subcostal nerve
B) T8
C) T11
D) T10
E) T9

41).In order to avoid injury to inferior epigastric vessel needle for paracentesis must pass through:
A) Lateral to mc Burney`s point
B) Medial to mc Burney`s point
C) Medial to Palmer`s point
D) Lateral to Palmer`s point
E) Umbilicus

40.Bifurcation of abdominal aorta present at level of plane of iliac crests which is at level of :
A) T12
B) L3
C) L4
D) T1

39.Transpyloric level indicates:


A) Pylorus of stomach
B) Duodenojejunal flexure
C) Fundus of gall bladder
D) Renal hila and neck of pancreas
E) All of the above
38.Transpyloric plane is usually present hand breath below xiphod process , at ninth coastal cartilage,
marks termination of spinal cord is correspond to vertebral level:
A) T12
B) L1
C) T11
D) T10
E) T9

37.Aortic opening in the diaphragm usually present at the level of


A) T12
B) T8
C) T11
D) T10
E) T9

36.Inferior vena caval opening in the diaphragm usually present at the level of :
A) T12
B) T8
C) T11
D) T10
E) T9

35.Oesophageal opening in the diaphragm usually present at the level of :


A) T12
B) T8
C) T11
D) T10

34.Aspiration or insertion of chest drain is usually done through:


A) Mid clavicular line 5th intercostal space by running needle or drain over the rib
B) Mid axillary line 5th intercostal space by running needle or drain over the rib
C) Mid clavicular line 5th intercostal space by running needle or drain under the rib
D) Mid axillar line 5th intercostal space by running needle or drain under the rib
E) Mid clavicular line 4th intercostal space by running needle or drain over the rib

33.True ribs present:


A) 1 to 7
B) 8 to 10
C) 11 to 12
D) 1 to 8
E) 8 to 12

32.Angle of Louis lies at the level of :


A) T1
B) T2
C) T3
D) T4
E) T5
31.Presenting diameter of fetal skull is smallest with following presentation:
A) Occipital presentation
B) Face presentation
C) Vertex presentation
D) Brow presentation
E) Both a and b

30.Vitamin d deficiency cause:


A) Rachitic pelvis
B) Android pelvis
C) Anthropoid pelvis
D) Platypelloid pelvis
E) Gynaecoid pelvis

29. Obstetric conjugate is:


a) 10.5cm
b) 12cm
c) 13cm
d) 15cm
e) 17cm

28.Spread of malignancy from pelvic viscera to vertebral venous plexus system via:
a) Lateral sacral vessels
b) Lumbar vein
c) Posterior intercostal
d) Vertebral vein
e) Inferior mesenteric vein

27.IVC pierces diaphragm at level of :


a) L1
B) L2
C) T12
D) T1
E)T8

26.Common iliac vein join to form IVC behind right iliac vein at :
A) L1
B) L2
C) L3
D) L4
E) L5

25.External ilia artery become femoral artery beyond


A) L1
B) L2
C) L3
D) Inguinal ligament
E) Sacroiliac joint

24.Common iliac artery divides into internal and external iliac artery at
A) L1
B) L2
C) L3
D) L4
E) Sacroiliac joint

23)Which of the following arteries is a branch of the internal iliac artery?


A)Cystic
B)Deep Circumflex
C)Femoral
D)Ovarian
E)Uterine

23.Terminal branches of aorta is


A) Right and left common iliac artery
B) Median sacral artery
C) Phrenic artery, suprarenal artery,
D) Renal artery and gonadal artery
E) Both a and b

22)The femoral triangle contains all of the following structures EXCEPT which one?
Deep inguinal lymph nodes
Saphenofemoral Junction
Femoral sheath
Femoral nerve
Genital branch of the genitofemoral nerve

21)The roof of the femoral triangle is formed by which structure


Fascia lata
Sartorius
Adductor longus
Femoral sheath
Transversalis fascia

20)The basic functional unit of the kidney is what?


Loop of Henle
Nephron
Bowmans Capsule
Glomerulus
Calyx

5.Lymphatic drainage of uterus and fallopian tube is:


a) Fundus to inguinal lymph node and para-aortic lymph node
b) Body of uterus to external iliac group of lymph node
c) Cervix external iliac, internal iliac and sacral group of lymph node
d) Fallopian tube and ovarian lymphatic passes to para-aortic lymph node
e) All of above

4.. False statement about uterus is:


a) Anteriorly uterovesical pouch of peritoneum
b) Ureter is situated 12 mm lateral to lateral vaginal fornix
c) Peritoneum covers whole of uterus
d) Uterine artery is a branch of internal iliac artery and it is lies above ureter and supply uterus at level of
isthmus
e) Ovarian artery is a branch of aort

False statement about uterus is:


a) In nulliparous women external Os is circular
b) In pregnancy cervix has consistency like nose
c) Child cervix is twice size of body of uterus
d) Isthmus is 1.5cm in length

wrong statement among following is:


a) Cervix project into anterior vaginal wall
b) Posterior vaginal wall is 10cm
c) Anterior to vagina is pouch of Douglas
d) Posterior to vagina is rectum and anal canal
e) Lateraly ureter and levator ani

21.Canal of nuck present in:


a) Labia majora
b) Labia minora
c) Inguinal canal
d) Obturator fossa
e) Femoral canal

20.Uterus pierce:
a) Uterosacral ligament
b) Broad ligament
c) Cardinal ligament
d) Levator ani
e) Pubocervical ligament

19.Which of following is not true support of uterus:


a) Uterosacral ligament
b) Broad ligament
c) Cardinal ligament
d) Levator ani
e) Pubocervical ligament
18.False statement about anal triangle is:
a) Medially levator ani and external anal sphincter
b) Fascia over obturator internus with Alcock`s canal
C)Roof levator ani
d) Floor skin and fat
e) Posteriorly urogenital perineum

The inguinal canal is reinforced posteriorly by which structure?


Conjoint tendon
Inguinal ligament
External oblique aponeurosis
Internal oblique fibres
Lacunar ligament

Regarding Levator Ani which of the following statements is TRUE?


Arterial blood supply is from inferior rectal artery
Arterial blood supply is from superior rectal artery
Arterial blood supply is from internal iliac artery
Arterial blood supply is from inferior gluteal artery
Arterial blood supply is from superior gluteal artery

What kind of epithelium lines the endocervix?


Columnar
Statified Squamous, non keratinised
Statified Squamous, keratinised
Cuboidal

Which of the following is contained within the deep perineal pouch?


Proximal portion of urethra
Fatty layer superficial fascia
Ischiocavernosus muscle
Bulbospongiosus muscle
Superficial transverse perineal muscle

You see a patient who is 32 weeks pregnant. She complains of tingling to the right buttock and shooting
pain down the leg. You suspect Piriformis syndrome. Regarding Piriformis which of thefollowing
statements are true?
Nerve supply is via S2 to S4
Origin is ischial tuberosity
Insertion is onto the greater trochanter
Main action is internal rotation of the hip
Arterial supply is primarily via the medial sacral artery

17.Which of following is not containt of superfascial perineal pouch :a) Ischiocavernosus muscle
b) Bulbospongiosus muscle
c) Superficial transverse perineal muscle
d) Deep transverse perineal muscle
e) Crura of clitoris
16.Levator ani is supplied by:
a) Pudendal nerve
b) Obturator nerve
c) Femoral nerve
d) Hypogastric nerve
e) Sciatic nerve

5.Which of following indicate pelvis is contracted:


a) Diagonal conjugate 12.5 cm
b) Sacral promontory can be easily reached
c) Transvers diameter 13 cm
d) Clinched fist should fit between ischial tuberosities
e) Pelvic walls concav

The MOTOR never supply to the uterus is via?


A. Sup hypogastric plexus
B. Inf hypogastric plexus
C. Pelvic splanchnic n
D. Pudendal n
MOTOR IS C AND SENSORY IS B

12.Aorta arises divide into right and left common iliac artery at:
A) L1
B) L2
C) L3
D) L4
E) T12

21.Inferior mesenteric artery a ventral branch of aorta arises at


A) L1
B) L2
C) L3
D) L4
E)T10

20.Superior mesenteric artery a ventral branch of aorta arises at


A) L1
B) L2
C) L3
D) L4
E) T12

19.Coeliac artery a ventral branch of aorta arises at


A) L1
B) L2
C) L3
D) L4
E) T12

18.Aorta enters abdomen at:


A) L1
B) L2
C) L3
D) L4
E) T12
17.Obliterated remains of Urachus called:
A) Median umbilical ligament
B) Medial umbilical ligament
C) Ductus arteriosus
D) Ligamentum arteriosum
E) Ligamentum venosum

16.Ductus venosus become:


A) Medial umbilical ligament
B) Foramen ovale
C) Ductus arteriosus
D) Ligamentum arteriosum
E) Ligamentum venosum

15.Oxygenated blood from umbilical vein to inferior vena cava is bypasses by:
A) Ductus venosus
B) Foramen ovale
C) Dctus arteriosus
D) Ligamentum arteriosum
E) Ligamentum venosum

14)Cervix and upper one third of vagina drain into:


A) Superficial inguinal lymph node
B) Deep inguinal lymph node
C) External iliac nodes
D) Internal iliac nodes
E) Obturator nodes

13.Vulva drain into


A) Superficial inguinal lymph node
B) Deep inguinal lymph node
C) External iliac nodes
D) Internal iliac nodes
E) Deep femoral lymph node of cloquet

12.Clitoris drain into :


A) Superficial inguinal lymph node
B) Deep inguinal lymph node
C) External iliac nodes
D) Internal iliac nodes
E) Obturator nodes

11.Cisterna chyli lies over


A) L1,L2
B) T12,L1
C) L2,L3
D) L4,L5
E) T11,T12

In the male urethra which is the narrowest part


Prostatic urethra
Membranous urethra
Penile urethra
Spongy urethra
Cavernous urethra
A baby with shoulder dystocia suffers a brachial plexus injury. You diagnose Erb-Duchenne palsy. Which
nerve roots are typically affected?
C3 and C4
C4 and C5
C5 and C6
C6 and C7
C7 and T1

The external anal sphincter is innervated by which nerves?


Inferior rectal
Superior rectal
Pelvic Splanchnic
Ilioinguinal
genitofemoral

What testicular cell type secretes testosterone?


Myoid
Intersitial macrophages
Sertoli
Leydig
Germ

The blood-testis barrier is formed by junctions between which type of cell?


Leydig
Germ
Sertoli
Myoid
Morgagni

You are called to see a women after a prolonged labour with failed instrumental delivery converted to c-
section. She is unable to dorsiflex her right foot and complains of pins and needles to the foot and lower
leg. What is the likely diagnosis?
Right L5 nerve root compression
Right S1 nerve root compression
Right Saphenous nerve root compression
Right common peroneal nerve root compression
Right superficial peroneal nerve root compressionNext question

70.Which of the following is the best description of the pelvic diaphragm?


(A) made up mainly by the coccygeus
(B) covered on one side by fascia and on the other by peritoneum
(C) a muscle innervated by L2, L3, and L4
(D) an extension of the sacrococcygeal ligament
(E) synonymous with the pelvic floor
Key is E
The pelvic diaphragm(also called the pelvic floor) is made up of the levator ani muscle and thecoccygeus. It is connected to the
pelvic side-wall by its attachment to the obturator internus muscle atthe arcus tendineus. The pelvic diaphragmprovides support
and closure for the intraperitoneal cavitycaudally just as the thoracic diaphragm provides closure in the cephalad direction. It is
covered byfascia on both sides and innervated fromS2, S3, S4. The potential spaces through which the vagina,urethra, and
rectumpass are the possible sites of pelvic prolapse

69.In a female, which of the following best describes the urogenital diaphragm?
(A) includes the fascial covering of the deep transverse perineal muscle
(B) encloses the ischiorectal fossa
(C) is synonymous with the pelvic diaphragm
(D) is located in the anal triangle
(E) envelops the Bartholin’s gland
Key is A
The urogenital diaphragmis immediately cephalad to the muscles of the external genitalia. Itconsists of a tough fibrous fasci al
membrane inferiorly covering the triangular area under the pubicarch and extending posteriorly to the ischial tuberosities. It is
penetrated by the urethra and vagina inthe female. Just cephalad to this fascia are the deep transverse perineal muscle and t he
urethralsphincter mechanism. The superior fascia of the urogenital diaphragmis attached tightly to these
muscles and is just caudad to the levator ani muscle. The urogenital diaphragm supplies support for theanterior vagina, ureth ra,
and trigone of the bladder. The area encompassing the urogen ital diaphragmand the superficial and deep perineal spaces is
referred to as the urogenital triangle.

Where are the proximal and distal centriole located in a spermatozoa?


Head
Neck
Middle Piece
Tail
End Piece

Regarding lymph drainage of the ovary where does the majority of lymph drain to?
Inguinal nodes
Mesenteric nodes
para-aortic nodes
internal iliac lymph nodes
External iliac lymph nodes
Which of the following statements regarding the Vaginal artery is typically TRUE?
It arises from the Internal iliac artery
It arises from the External iliac artery
It arises from the Abdominal Aorta
It arises from the Ovarian artery
It arises from the Uterine artery

The testis receive innervation from which spinal segment


T10
T12
S1
S2
S3

The anterior scrotal artery is a branch of which artery?


Internal Pudendal
External Pudendal
Cremasteric
Testicular
Gonadal

The lower part of the rectum is supplied by the middle rectal artery. What is the middle rectal artery a
branch of?
Inferior mesenteric artery
Superior mesenteric artery
Internal iliac artery
External iliac artery
Internal pudendal artery

Regarding the structure of the detrusor muscle. Which of the following is true?
The detrusor is a single layer of longitudinal smooth muscle
The detrusor is a single layer of circular smooth muscle
The detrusor is a single layer consisting of both circular and longitudinal muscle fibres
The detrusor is divided into 2 layers consisting of an inner longitudinal smooth muscle layer and outer
circular smooth muscle layer
The detrusor is divided into 3 layers consisting of inner and outer layers of longitudinal smooth muscle
with a middle circular smooth muscle layer

Which of the following best describes the mechanism of action of Prochlorperazine?


Dopamine D1 receptor antagonist
Dopamine D2 receptor antagonist
Histamine H1 receptor antagonist
Histamine H2 receptor antagonist
5HT3 receptor antagonist

Which of the following arteries is a branch of the internal iliac artery?


Cystic
Deep Circumflex
Femoral
Ovarian
Uterine

The arcuate line forms part of the border of the pelvic brim. Where is it located?
ilium
sacrum
pubis
Ischium

The hypothalamus is located in which part of the brain?


Midbrain
Diencephalon
Brain stem
Corpus callosum
Mesencephalon

1.The median umbilical ligament is a remnant of what structure?


Umbilical veins
Paraumbilical Veins
Umbilical arteries
Paraumbilical arteries
Urachus

The lymph from the scrotum drains into which nodes


Lumbar nodes
Para-Aortic nodes
Hypogastric nodes
Deep Inguinal nodes
Superficial Inguinal
nodes

The axial filament of the sperm tail has what type of arrangement of filaments?
1+5
1+7
3+5
7+5
9+2

The inferior 1/3 of the rectum is principally supplied by which artery?


Superior rectal artery
Middle rectal artery
Inferior rectal artery
Sigmoid artery
External iliac

The ureter is lined by what type of epithelium?


Stratified
Pseudostratified
Transitional
Simple Cuboidal
Simple Columnar

You are asked to assess a patients perineal tear following labour by vaginal delivery. You note a
laceration that extends approximately a quarter of the thickness through the external anal sphincter.
How would you classify this tear?
1st
2nd
3a
3b
3c

30.True statement among following is:


a) Mons pubis is made up of adipose tissue and cover with hair and sebaceous gland
b) Cleft between labia majora is termed as vestibule
c) Ischiocavernosus muscle cut during medio-lateral episiotomy
d) Clitoris contain pair of carpora cavernosa
e) Bulb of vestibule lies on each side of ischiocavernosus muscle

Regarding Cremaster muscle which is true ...


A..arises from external oblique
B.arises from internal oblique
C arises from fascia of tranversus abdominis
D.is supplied by branches of femoral artery.

onexamination
30.Which one of the following options describes the lymph group to which the ovaries drain?
A Deep inguinal lymph nodes
B Internal iliac lymph nodes
C Para‐aortic lymph nodes
D Superficial inguinal lymph nodes
E Superior mesenteric lymph nodes

29.Which one of the following options describes the lymph group to which the distal rectum drains?
A Deep inguinal lymph nodes
B Internal iliac lymph nodes
C Para‐aortic lymph nodes
D Superficial inguinal lymph nodes
E Superior mesenteric lymph nodes

28.Which one of the following options describes the lymph group to which the base of the cervixdrains?
A Deep inguinal lymph nodes
B Internal iliac lymph nodes
C Para‐aortic lymph nodes
D Superficial inguinal lymph nodes
E Superior mesenteric lymph nodes
27.Which of the following options describes the lymph group to which the vulva drains?
A Deep inguinal lymph nodes
B Internal iliac lymph nodes
C Para‐aortic lymph nodes
DSuperficial inguinal lymph nodes
E Superior mesenteric lymph nodes

26.Which of the following options describes the lymph group to which the big toe drains?
A Deep inguinal lymph nodes
B Internal iliac lymph nodes
C Para‐aortic lymph nodes
DSuperficial inguinal lymph nodes
E Superior mesenteric lymph nodes

25.Which of the following nerves supply the ankle reflex?


A C7, C8
B C8, T1
C C8, T1
D L1
E L5, S1

24.Which of the following nerves supply the knee reflex?


A C5, C6
B C7, C8
C C8, T1
D L1
E L3, L4 femoral n

23.The biceps jerk is elicited by


A.C5, C6
B C7, C8
C C8, T1
D L1
E L3, L4

22.Which one of the following nerve roots mediates the abdominal reflex?
A C5, C6
BT7, T8, T9
C L1
D L3, L4
E L5, S1

21.Which one of the following nerve roots mediates the anal reflex?
A C5, C6
B C7, C8
C C8, T1
D L5, S1
E S3, S4

20.Which one of the following branches of the abdominal aorta arises from the aorta between thelevel
of L1 and L2 vertebrae, and forms five segmental arteries that do not anastomose?
A Inferior mesenteric artery
B Lumbar arteries
C Median sacral artery
D Renal arteries
E Superior mesenteric artery

19.Which one of the following branches of the abdominal aorta is an unpaired branch that arisesfrom
the aortic bifurcation?
A Inferior mesenteric artery
B Lumbar arteries
C Median sacral artery
D Renal arteries
E Superior mesenteric artery

18.Which one of the following branches of the abdominal aorta gives rise to the left colic, sigmoid and
superior rectal arteries?
A Inferior mesenteric artery
B Lumbar arteries
C Median sacral artery
D Renal arteries
E Superior mesenteric artery

17.Which one of the following branches of the abdominal aorta gives rise to the intestinal, middlecolic
and right colic arteries?
A Inferior mesenteric artery
B Lumbar arteries
C Median sacral artery
D Renal arteries
E Superior mesenteric artery

16.Which one of the following branches of the abdominal aorta gives rise to the left gastric, splenicand
hepatic arteries?
A Adrenal arteries
B Coeliac trunk
C Common iliac arteries
D Gonadal (ovarian or testicular) arteries
E Inferior mesenteric artery

15.Which one of the following dermatomal level corresponds to the innervation of the anus?
A S1
B S2
C S3
D S4
E S5

14.Which one of the following dermatomal levels innervates the fifth digit of the foot?
A L4
B L5
C S1
D S2
E S3

13.Which one of the following dermatomal levels innervates the knee?


A L1
B L2
C L3
D L4
E L5

12.Which paired structures contain the uterine arteries and veins?


A Broad ligaments
B Cardinal ligaments
C Pubocervical ligaments
D Round ligaments
E Uterosacral ligaments

11.Which vessels anastomose with the vaginal arteries to form the blood supply to the lower third of the
vagina?
A Ovarian artery
B Spiral arteries
CTerminal branches of internal pudendal artery
D Uterine artery
E Vaginal branch of uterine artery

10.From which embryological structure is the round ligament of the uterus derived?
A Cloaca
B Gubernaculum
C Mullerian duct
D Urachus
E Wolffian duct

9.What is the lymphatic drainage of the ovaries?


A Common iliac nodes
B External iliac nodes
C Internal iliac nodes
D Para‐aortic nodes
E Sacral nodes

8.Into which of the following blood vessels does the left ovarian vein drain?
A Anastomosis with the uterine vein
B Inferior vena cava
C Labia minora
D Left renal vein
E Superior mesenteric vein

7.Which one of the following structures is the termination of the round ligament?
A Deep inguinal ring
B Labia majora
C Labia minora
D Lateral vaginal walls
E Superficial inguinal ring

6.What is the average length of the newborn in the United Kingdom?


A 40 cm
B 42 cm
C 46 cm
D 50 cm
E 54 cm

5.After discussion with her family and yourself she has decided that she would like a termination.She is
terrified of having an operation and requests a medical termination.
You explain the process and give her mifepristone.What is the action of this drug?
A Imitates GnRH
B Imitates oxytocin
C Inhibits HCG
D Inhibits the action of oestrogen
EInhibits the action of progesterone

4.A 76 year old lady present with painless frank haematuria.A cystoscopy identified blader carcinoma.u
organized CT for tumor staging
To which lymph nodes are metastases most likely?
A External iliac
B Deep inguinal
C Para aortic
D Popliteal
E Superficial inguinal

A.simple columnar
B.simple cubidal
C.simple squamous
D.stratified squmaous
E.stratified columnar
2.Which of the following nerve roots, if compressed by a herniated disc, would result in adiminished
knee jerk?
A L2
B L4
C L5
D S1
E S2

1.Which of the following is not a branch of the pudendal nerve?


A Perineal nerve
B Dorsal nerve of the penis
C Inferior rectal nerve
D Genitofemoral nerve
E Posterior scrotal nerv

The MOTOR never supply to the uterus is via?


A. Sup hypogastric plexus
B. Inf hypogastric plexus
C. Pelvic splanchnic n
D. Pudendal n

68.Pelvic inflammatory disease (PID) occurs in women because of which of the following
characteristicsof the fallopian tube?
(A) It is a conduit fromthe peritoneal space to the uterine cavity.
(B) It is found in the utero-ovarian ligament.
(C) It has five separate parts.
(D) It is attached to the ipsilateral ovary by the mesosalpinx.
(E) It is entirely extraperitoneal.
Key is A
Fallopian tubes are a conduit fromthe peritoneal to the uterine cavity, which can also allowspermor bacteria fromthe vagina
through the uterus to the peritoneal cavity. Each tube is covered by
peritoneumand consists of three layers: serosa, muscularis, and mucosa. They tr averse the superiorportion of the broad ligament
attached by a mesentery (mesosalpinx). It has four distinct areas in its 8 -to 12-cmlength: the portion that runs through the uterine
wall (interstitial or cornual portion), theportion immediately adjacent to the uterus (isthmic portion), the midportion of the tube
(ampulla), andthe distal portion containing the finger-like fimbriae that sweep the ovuminto the infundibu -lumof thetube. The
fimbriae are intraperitoneal. The tubal lumen becomes increasingly more complex as itapproaches the ovary. In tubal
reanastomoses, the greatest success is attained when isthmic-isthmic oristhmic-ampullary regions can be reapproximated. The
longest of the fimbriae (the fimbriae ovarica) isattached to the ovar

67.A patient presents approximately 10 years post-menopausal with complaints of pressure vaginally
andthe sensation that something is falling out. When told she has a fallen uterus, she wonders if it is due
tothe damage fromher round ligaments since she had a great deal of round ligament pain during
herpregnancies. Which of the following ligaments provide the most support to the uterus in terms of
preventing prolapse?
(A) broad ligaments
(B) round ligaments
(C) utero-ovarian ligaments
(D) cardinal ligaments
(E) arcuate ligament
Key is D
The cardinal ligaments are also called the transverse cervical ligaments, or Mackenrodt’sligaments, and are considered part o f the
uterosacral ligament complex. These ligaments serve as themajor support for the apex of the vagina and are seve red at the time of
hysterectomy. Once divided at
hysterectomy, vaginal vault prolapse becomes more likely. The broad ligaments are mainly peritoneum
and the round ligaments mainly muscle. Neither provides much support. The arcuate ligament is notattached t o the uterus.

66.The uterus and adnexa have some relatively fixed anatomic characteristics that can be noted on
pelvicexamination or laparoscopic observation. Which of the following characteristics would you most
likely
find in a normal patient?
(A) retroflexion of the uterus
(B) ovaries caudad to the cervix
(C) round ligaments attached to the uterus posterior to the insertion of the fallopian tubes
(D) immobility of the uterus
(E) cervix not palpable on rectal examination
Key is A
The cervix protrudes into the fornix of the vagina, and the ovaries are intraperitoneal; therefore,they are
found cephalad to the cervix. The round ligaments are attached to the uterus anterior to theattachment
of the fallopian tubes. Retroflexion implies a sharp angle between the cervix and the fundus
of the uterus, which is bent posteriorly. This is a less common position of the uterus, which can
also,more commonly, be midposition or anteflexed. These are all normal positions of the uterus. It is
important to recognize which way the uterine body is flexed so that you do not perforate the
loweruterine segment while sounding the uterus or dilating the cervix. The uterus is normally mobile
and if it
is not, adhesions or tumor may be present. The cervix is normally palpated anterior to the
rectumonrectal examination

65.What is the uterine corpus mainly composed of?


(A) fibrous tissue
(B) estrogen receptors
(C) smooth muscle
(D) elastic tissue
(E) endometrium

64.How do nabothian cysts occur?


(A) Wolffian duct remnants
(B) blockage of crypts in the uterine cervix
(C) squamous cell debris that causes cervical irritation
(D) carcinoma
(E) paramesonephric remnants

63.In the uterus of a normal female infant, what is the size relationship of the cervix, isthmus, and
fundus?
(A) The cervix is larger than the fundus.
(B) The isthmus is longer than either the cervix or the fundus.
(C) They are of equal size.
(D) The fundus is the largest portion.
(E) The cervix is smaller than either the isthmus or the fundus.
62.The clitoris is a major sensory sexual organ. Where does it get its major nerve supply from?
(A) lumbar spinal nerve
(B) pudendal nerve
(C) femoral nerve
(D) ilioinguinal nerve
(E) anterior gluteal nerve
Key is B
The clitoris consists of two crura, a short body, and the glans clitoris with overlying skin calledthe prepuce. It is attache d to the
pubic bone by a suspensory ligament. Within the shaft are corporacavernosa consisting of erectile tissue (loose in structu re) that
engorges with blood, causing erectionand enlargement (two times usual size) during sexual excitement. The clitoris and prepuc e
are the
primary areas of erotic stimulation in most women. The prepuce has the most innervation, which
usually comes froma terminal branch of the pudendal nerve in most women. Some women, however,have alternate innervations
and, in a few, innervation is sparse

61.During a physical examination myrtiform caruncles may be noted. What are they?
(A) circumferential nodules in the areola of the breast
(B) healing Bartholin’s cysts
(C) remnants of the Wolffian duct
(D) remnants of the hymen
(E) remnants of the Müllerian duct
Key is D
The hymen is a membrane that may cover all or part of the vaginal opening just above thevestibule. It may vary frombeing only
small integumental remnants (known as myrtiformcaruncles) tobeing perforated with one or many openings of various sizes, to
being completely closed (imperforate hymen) and require surgical intervention to allow menstruumto drain . The presence of
myrtiformcaruncles is not pathognomonic of prior vaginal penetration (e.g., intercourse or childbirth). They areof no patholo gic
significance.

60.During the performance of a pelvic examination, the area of the Bartholin’s ducts should be
inspected.Where do the Bartholin’s glands’ ducts open?
(A) into the midline of the posterior fourchette
(B) bilaterally, beneath the urethra
(C) bilaterally, on the inner surface of the labia majora
(D) bilaterally, into the posterior vaginal vestibule
(E) bilaterally, approximately 1 cmlateral to the clitoris
Key is D
The vestibule is an area enclosed by the labia minora. Bartholin’s glands, sometimes called themajor vestibular
glands, open into the posterior vestibule. These glands are prone to infection with
resulting occlusion of the ducts and the formation of grossly enlarged tender cysts .

59.The shape of the escutcheon may change with masculinization. The presence of a male escutcheon in
afemale is one of the clinical signs of hirsutismor increased testosterone. What is the usual shape of
theescutcheon in the normal female?
(A) diamond shaped
(B) triangular
(C) oval
(D) circular
(E) heart shaped
Key is B
The escutcheon, or configuration of the pubic hair on the mons veneris and lower abdomen, isgenerally an inverted
triangle in the female. It is considered asecondary sex characteristic. The malepat tern (a diamond shape extending
upward toward the umbilicus) may exist in 25% of women.Some times a male-pattern escutcheon in the female may
be associated with increased levels ofandrogens .

58. Under the influence of relaxin and the pressure of pregnancy the junction between the two pubic
bonesmay become unstable near the time of delivery. This will result in a waddling gait in the woman
tominimize discomfort. What is this junction called?
(A) sacroiliac joint
(B) symphysis
(C) sacrococcyg
(D) piriformis
(E) intervertebral joint
Key is B
The joint between the two pubic bones is the pubic symphysis. It is not a stable joint. Jointsbetween the bones of the pelvis , such
as the sacroiliac and sacrococcygeal, are called synarthroses.
They have limited motion but do become more mobile and even separate a bit during pregnancy. Therelaxation is attributed to
the hormone relaxin. The piriformis is a muscle.

57. The plane fromthe sacral promontory to the inner posterior surface of the pubic symphysis is
animportant dimension of the pelvis for normal delivery. What is the name of this plane?
(A) true conjugate
(B) obstetric conjugate
(C) diagonal conjugate
(D) bi-ischial diameter
(E) oblique diameter
Key is B
The obstetric conjugate is the shortest line fromthe inside of the symphysis to the most prominentpoint on the front two segments
of the sacrum. It defines what is often the smallest diameter of the
pelvic inlet. It should be estimated during clinical examination (pelvimetry) and considered wheneverevaluating a pelvis for
possible cephalopelvic disproportion, especially during abnormalities of labor .
It differs fromthe true conjugate, which is measured fromthe top of the symphysis, and also fro mthediagonal conjugate, which is
measured clinically fromthe bottomof the symphysis to the sacral
promontory. The biischial diameter is on the pelvic outlet.

56. The part of the pelvis lying above the linea ter-minalis has little effect on a woman’s ability to
delivera baby vaginally. What is the name of this portion of the pelvis?
(A) true pelvis
(B) midplane
(C) outlet
(D) false pelvis
(E) sacrum
Key is D
The false pelvis or pelvis major lies above the linea terminalis. It seldomaffects obstetricmanageme nt, and
measurements of the iliac crest flare do not usually aid in determining the size of thetrue pelvis. An important
measurable indicator of the size of the true pelvis is the inter -spinousdiamete

55. During normal delivery, an infant must pass through the maternal true pelvis. Which of the
followingmost accurately describes the characteristics of the true pelvis?
(A) It has an oval outlet.
(B) It has three defining planes: an inlet, a midplane, and an outlet.
(C) It has an inlet made up of a double triangle.
(D) It is completely formed by two fused bones.
(E) It lies between the wings of the paired ileum.
Key is B
The true pelvis has three planes: inlet, mid-plane, and outlet. It is made up of the paired ileum,ischium, and pubic
bones, and the single sacrumand coccyx. The true pelvis is cau-dad to the false
pelvis, which lies between the paired ileumwings. Its inlet is usually gynecoid .

54. The human pelvis is a complex structure that permits upright posture and being capable with
childbirthdespite the relatively large fetal head. Which option includes all of the bones that make up the
pelivs?
(A) trochanter, hip socket, ischium, sacrum, and pubis
(B) ilium, ischium, pubis, sacrum, and coccyx
(C) ilium, ischium, and pubis
(D) sacrum, ischium, ilium, and pubis
(E) trochanter, sacrum, coccyx, ilium, and pubis
Key is B
The pelvis surrounds the birth passage, provides attachment for muscles and fascia, and includesthe ilium, ischium,
pubis, sacrum, and coccyx. The ilium, ischium, and pubic bone compo se the
innominate bone.

53. The inguinal canal in an adult female was opened surgically. Which of the following structures
wouldnormally be found?
(A) a cyst of the canal of Nuck
(B) Gartner’s duct cyst
(C) Cooper’s ligament
(D) the round ligament and the ilioinguinal nerve
(E) the pyramidalis muscle
Key is D
The superficial inguinal ring is just cepha-lad to the pubic tubercle and just lateral to it, the deepinguinal ring passes through the
transver-salis fascia. The connection of these rings forms the inguinal
canal. The round ligament, the ilioinguinal nerve, and the processus vaginalis pass out of the abdomenthrough this canal (as does
the spermatic cord in the male). Gartner’s ducts are found in the lateral
walls of the vagina. One would not normally find a cyst of the processus vaginalis (cyst of the canal of
Nuck).

52. Hernias occur more commonly in men than in women beneath the thickened lower margin of a
fascialaponeurosis extending fromthe pubic tubercle to the anterior superior iliac spine. This thickened
fascia
is called which of the following?
(A) inguinal ligament
(B) Cooper’s ligament
(C) linea alba
(D) posterior rectus sheath
(E) round ligament
Key is A
Fromthe pubic tubercle to the anterior superior iliac spine, the thickened lower margin of thefas cial aponeurosis forms the
inguinal ligament. This aponeurosis of the external oblique muscle fuses
with its counterpart fromthe opposite side and with the underlying internal oblique fascia. Cooper’sligament is a thickening of
fascia along the pubic bone. The linea alba is in the midline and the roundligament attaches to the uterus.

51. A healthy 5 ft 6 in. tall, adult female is most likely to have a pelvic inlet that would be classified
aswhich of the following Caldwell-Moloy types?
(A) android
(B) platypelloid
(C) anthropoid
(D) gynecoid
(E) triangular
Key is D
Pelvises in most U.S. women are gynecoid, but they may be of a mixed type (for instance, having agynecoid
forepelvis and an anthropoid posterior pelvis). The obstetrician has to judge the capacity ofthe pelvis on the basis of
its total configuration, including midplane and outlet capacities, and always inrelation to the size and position of the
fetus.

50.Which of following is not difference between male and female pelvis visible on x –ray:
a) Heart shaped in male and oval in female
b) Pubic angle is narrow in male and wide in female
c) Soft tissue shadow scrotum is seen in female
d) Anteroposterior diameter is more in female
e) Sacrum is wide in female

49.Most powerfull ligament in body is:


a) Anterior sacroiliac ligament
b) Posterior sacroiliac ligament
c) Sacrotuberous ligament
d) Sacrospinous ligament
e) Anterior cruciate ligament

48.False statement among following is:


a) Fusion of fifth lumber vertebrae with sacrum is called lumberization of sacral vertebrae
b) Separation of first sacral vertebrae from rest of sacrum is called lumberization of sacral vertebrae
c) Dural sheath terminate at level of S2
d) Coccyx is made up of 5 diminitive vertebrae
e) Symphysis pubis is cartilaginous joint

47.Sure sign for sacral hiatus is:


a) Sacral cornu
b) Sacral promontory
c) C7 spine
d) Ischial tuberosity
e) Ischial spine

46.Sacral hiatus transmits:


a) S4
b) S5
c) S1
d) S2
e) All of above

45.True and false pelvis is separated by:


a) Inferior rami of pubis
b) Ischial tuberosity
c) Iliopectineal line
d) Both a and b
e) Ischial tuberosity

44.False statement about Pfannensteil`s incision is


a) Given 2 cm above pubic symphysis
b) Peritoneum is separated superiorly
c) Rectus sheath is separated from rectus muscle upto umbilicus above
d) Rectus sheath separated from rectus muscle upto pubic symphysis below
e) Bladder must be empty

43. Suprapubic skin is supplied by:


a) Iliohypogastric nerve
b) Genitofemoral nerve
c) Ilioinguinal
d) Femoral
e) Obturator

42. Spinal anesthesia is given for elective LSCS, surgeon


check for level of anesthesia by at level of umbilicus.
Umbilicus correspond to
a) T7
b) T10
c) L1
d) T4
e) L4

41.Anterior abdominal wall is innervated by:


a) Posterior primary rami of t7 to l1
b) Anterior primary rami of t7 to l1
c) Anterior primary rami of t9 to l1
d) Posterior primary rami of t9 to l1
e) None of above

40.In Lerich syndrome anastomosis between superior and


inferior epigastric artery connects:
a) Subclavian and internal iliac artery
b) Subclavian and external iliac artery
c) Femoral and internal iliac artery
d) Femoral and external iliac artery
e) None of above

39. Inguinal ligament which extend from superior iliac


spine to pubic tubercle represents:
a) Rolled out lower border of external oblique
aponeurosis
b) Rolled out lower border of internal oblique
aponeurosis
c) Rolled out lower border of transverse abdominus
d) Both a and b
e) All of above

38.Posterior rectus sheath is absent in:


a) Below arcuate line
b) Over 5 to 7 coastal cartilage
c) Between umbilicus to pubic crest
d) Above arcuate line
e) Both a and b

37.Function of anterior abdominal wall muscle are:


a) Flex trunk
b) Act as accessory muscle for respiration
c) Help in defecation, micturition, parturation
d) Protect abdominal organ from trauma
e) All of above

36.What artery supplies the distal third of the transverse colon?


A Celiac trunk artery
B.Inferior mesenteric artery
C.Left renal artery
D.Right renal artery
E.Superior mesenteric artery

35.What nerve supplies to the anterior one-third of the labium majus?


A.Dorsal branch of the pudendal nerve
B.Inferior rectal nerve
C.Ilioinguinal nerve
D.Obturator nerve
E.Pudendal nerve

34 What part of the spermatozoa has the highest concentration of mitochondria?


A.Acrosome
B.Flagellate tail
C.Middle piece
D.Principle piece
E.True head

33.Which testicular cells are located outside the blood/testis barrier?


A.Leydig
B.Primary spermocytes
C Sertoli
D.Spermatids
E.Spermatogonia

32.What type of cells are sertoli cells as defined by shape?


A.Circular cells
B.Cubodial cells
C.Epithelia with gap junctions
D.Flat columnar
E.Tall columnar

31.What is the layer between the theca cells and the mural granulosa?
A.Basal lamina
B.Basement membrane
C.Podocysts
D.VEGF unregulated capillary bed
E.Zona pellucida

30.With regard to ovulation, what is the fluid filled space in a follicle called?
A.Antrum
B.Basal lamina
C.Blastocyst
D.Lacuna
E.Uniblastocyst

29.what size is resting follicle


A.0.02 mm
B. 2 mm
C.10 mm
D.15 mm
E.20 mm

28.What size is a follicle that is ready to ovulate?


A.2 mm
B.4 mm
C.10 mm
D.15 mm
E.20 mm

27.Regarding the urinary bladder, which of the following is correct?


A.A glycosaminoglycan layer coats the luminal surface of the bladder mucosa
B.At the trigone, the epithelium is loosely adherent to the underlying muscle
C.The bladder wall consists of two layers of detrusor muscle
D.The uterovesical fold creates the Pouch of Douglas
E.The urinary bladder is covered entirely by peritoneum

26.The internal pudendal artery leaves the pelvis via what structure?
A.Greater sciatic foramen
B.Lesser sciatic foramen
C.Obturator foramen
D.Pudendal canal
E.Sacral canal

25.A direct inguinal hernia that develops from the weakness of the conjoint tendon can cause
damage to what nerve?
A.Genitofemoral nerve
B.Iliohypogastric nerve
C.Ilioinguinal nerve
D.Pudendal nerve
E.Subcostal nerve

24.Lymphatic drainage of the uterus is primarily to:


A.Common iliac nodes
B.External iliac nodes
C.Internal iliac nodes
D.Para-aortic nodes
E.Superficial inguinal nodes

23.From which embryological structure does the round ligament of the uterus originate?
A.Cloaca
B.Gubernaculum
C.Mullerian duct
D.Urachus
E.Wolffian duct

22.Which of the following is NOT a branch of the anterior division of the internal iliac artery?
A.Ilieolumbar artery
B.Internal pudendal artery
C.Obturator artery
D.Uterine artery
E.Vaginal artery

21.What is the lymphatic drainage of the ovaries?


A.Common iliac nodes
B.External iliac and superficial iliac nodes via the round ligament
C.External iliac nodes
D.Internal iliac nodes
E Lateral aortic and preaortic node

20.A 30-year-old woman has developed haematuria, pain and reduced urine output on day 3
following a total abdominal hysterectomy. A urine dipstick showed red blood cells but no nitrites. An
X-ray of the pelvis was unremarkable. What is the most likely diagnosis?
A.Bladder injury
B.Intra-abdominal bleeding
C.Ureteric stone
D.Ureteric trauma
E.Urinary tract infection

19.u have been asked to suture a deep episiotomy by a midwife. On examination you notice that the
external anal sphincter is mostly torn, with a few fibres left intact. The internal anal sphincter is intact.
How would this tear be described?
A.Second-degree tear
B.Third-degree tear, 3a
C.Third-degree tear, 3b
D.Third-degree tear, 3c
E.Fourth-degree tear

True about inferior mesentric artery:


A.Continues as inferior rectal artery
B.Takes part in formation of marginal artery
C.Arises from coeliac trunk
D.Supplies mid gut
E.Hepatic artery is a branch from it.

Pain of ovarian pathology referred to:


A.Back of thigh.
B.Anterior Thigh
C.Medial Thigh
D.Gluteal region
E.Umblical region

Gartner's duct is present in:


A.Vaginal wall
B.Broad ligament of uterus.
C.Transverse cervical ligamnet
D.Perineal body
E.Utero vesical pouch.

:Where is the "Cave of Retzius" present?


A.Between urinary bladder and rectum.
B.Between urinary bladder and cervix
C.Infront of bladder
D.Between cervix and rectum.

9.The lower third of the vagina has lymphatic drainage to which nodes?
Common iliac nodes
External iliac nodes
Internal iliac nodes
Superficial inguinal nodes
Para-aortic nodes

Which of the following nerves pierces the internal oblique muscle and passes through the inguinal
canal:
A. Iliohypogastric
B.Ilioinguinal
C. Genital branch of the genitofemoral
D.Obturator

Which structure is the principle supports of the uterus:


A. Broad ligament
B. Round ligament
C. Pelvic diaphragm
D. Transverse cervical ligament
E. Urogenital diaphragm

The following does not contribute to the boundaries of the ovarian fossa:
A Ureter
B External iliac vein
C Internal iliac artery
D Internal pudendal artery
E Obliterate umbilical artery
ANSWER (D)Internal pudendal artery
-It is bound by the external iliac vessels above and the internal iliac vessels and ureter behind.
-The obturator nerve crosses the floor of the fossa.
-The internal pudendal artery leaves the pelvis through the greater sciatic foramen.
-The medial umbilical ligaments arise from the superior vesicle arteries.
-The obliterated umbilical arteries run in front of the ovarian fossae

The cervix:
A Consists chiefly of smooth muscle
B Has a supravaginal part which is related anteriorly to the ureter
C Has a supravaginal part which is covered with peritoneum anteriorly
D Has pain sensation carried by the pelvic splanchnic nerves
E Is lined in its vaginal part by keratinized epithelium
-It has a higher fibrous tissue: smooth muscle ratio than the uterus.
- The ureters run within 2cm of the cervix on either side.
-It is separated anteriorly from the bladder by parametrium.
- In its vaginal part it’s covered by non-keratinized stratified squamous epithelium.
-The pelvic splanchnic nerves carry efferent parasympathetic fibres to the pelvic viscera.

The anal canal:


A Has an upper part which is innervated by the superior hypogastric plexus
B Has a lower part which is supplied by the superior rectal artery
C Drains lymph to the superficial inguinal nodes from its upper part
D Has its internal sphincter innervated by the inferior rectal nerve
E Has a superficial part of its external sphincter attached to the coccyx
ANSWER (E) Has a superficial part of its external sphincter attached to the coccyx
-The upper half of the anal canal is lined by columnar epithelium and has its nerve supply derived from the
autonomic hypogastric plexuses.
-The arterial supply is from the superior rectal artery, a branch of the inferior mesenteric artery, and the
lymphatic drainage is to the inferior mesenteric nodes.
-The lower half is lined by stratified squamous epithelium, which is innervated by the inferior rectal nerve.
- Its blood supply is from the inferior rectal artery and its lymph drains to the medial group of superficial
inguinal lymph nodes.
-The internal sphincter is innervated by sympathetic fibres from the inferior hypogastric plexus.
-The inferior rectal nerve (from the pudendal nerve) supplies the external anal sphincter, as does the perineal
nerve from S4.
-The superficial part of the external anal sphincter is attached to the coccyx behind and the perineal body in
front.

Concerning the rectus sheath:


A Above the costal margin rectus abdominis lies on the costal cartilages
B Below the arcuate line the internal oblique splits to enclose rectus abdominis
C It contains the musculophrenic artery
D It is innervated by the ilioypogastric nerve
E Has a fascia on its pelvic surface which gives origin to the levator ani
*****************************
ANSWER: (A) Above the costal margin rectus abdominis lies on the costal cartilages
-Rectus abdominis attaches to the xiphoid process, and 5th, 6th and 7th costal cartilages.
-The internal oblique splits to enclose rectus abdominis above the arcuate line.
-Below the arcuate line the fascia transversalis lies behind the rectus abdominis.
- The musculophrenic artery, a branch of the internal thoracic, runs around the costal margin of the diaphragm
and supplies the lower intercostals spaces and the diaphragm.
-The levator ani originates from the body of the pubis, fascia of the obturator internus and ischial spine.

False regarding the pituitary gland:


A Lies below the diaphragme sellae
B Is developed from two primordia
C Communicates with the hypothalamus
D Lies inferior to the optic chiasma
E Is anterior to the sphenoidal sinus
ANSWER (E) Is anterior to the sphenoidal sinus
- The hypophysis cerebri/Pituitary lies below the diaphragme sella, which separates the anterior lobe
from the optic chiasma.
-It lies above the body of the sphenoid (and sphenoid sinus), medially to the cavernous sinus and
anteriorly to the dorsum sellae, the basilar artery and the pons.
-The pituitary is formed formed of the adenohypophysis (anterior) and neurohypophysis (posterior),
which is a down growth of the diencephalon.
-The optic chiasma lies anterior to the pituitary stalk, which connects it to the hypothalamus.

Concerning the female reproductive system


A. The mesometrium suspends the ovary
B. The cardinal ligament provides a major support for the uterus
C. The rectouterine pouch is the shallow part of the pelvis
D. Uterus drains to external iliac vein
E. The uterosacral ligament attach to the lateral part of the sacrum.

C
A

A 21 year old woman under goes a laproscopic ovarian cystectomy to remove a dermoid cyst .three
days after the operation she presents to the emergency department feeling un well and her
haemoglobin level is found to be 6 g/dl . Damage to a blood vessel is suspected from the
laparoscopic procedure
Which vessel cross the common and external illiac artery in the infudibulopelvic fold?
A.Femoral artery
B.Inferior mesentric artery
C.Median sacral artery
D.Ovarian artery
E.Renal artery

The internal anal sphincter is innervated by which nerves?


A Inferior rectal
B Superior rectal
C Pudendal
D Pelvic Splanchnic
E Perineal

The femoral triangle is bounded superiorly by which of the following structures?


A Inguinal canal
B Conjoint tendon
C Inguinal ligament
D Sartorius

A 63 year old woman complains of numbness over her thigh following a radical hysterectomy for
stage lV endometrial carcinoma
What is the nerve root of the obturator nerve?
A.Anterior division L1-L4
B.Anterior division L2-L4
C.Anterior divisionL3-L4
D.Posterior division L2-L4
E.posterior divisionL3-L4

A 47 - year old woman under goes a routine trabsabdominal hysterectomy to remove a large fibroid
uterus. She is found to have a fibroid in the broad ligament and there is concern that her ureter may
have been damaged due to the difficult operation. With regards to the path of the ureter, which of the
following is correct?
A.In the broad ligament, both ureters pass over their respective uterine artery
B.Runs laterall to the internal illiac artery
C.Ovarian vessels enter the pelvic posterior to the ureters
D.Upper one third of the ureters lie in the abdomen
E.Ureters cross close to the bifurcation of the common illiac vessels

Superficial inguinal lymph nodes drains from all except


A) urethra
B) anal canal below the pectinate line
C) glans Penis
D) perineum
c drains to deep inguinal

The GP has referred a 76-year-old woman complaining of a lump 'down below'. She also has
backache, and a dragging sensation. On examination, she has a normal sized uterus and first
degree uterine descent. She is unsure of having surgery and would like to try conservative measures
first.
Which of the following structures prevents prolapse of the uterus and vagina?

Broad ligaments
Cardinal ligaments
Cooper's ligament
Fascia of Colles
Round ligament

Regarding Levator Ani which of the following statements is TRUE?


A) Innervation is derived from L5/S1 spinal nerves
B) Innervation is derived from S1/S2 spinal nerves
C) Innervation is derived from S3/S4 spinal nerves
D) Is supplied by the superior rectal nerve
E) Is supplied by the inferior gluteal nerve

Which one of the following branches of the abdominal aorta arises from the aorta between the level
of L1 and L2 vertebrae and forma 5 segmental arteries that do not anastomose?
A. Inf mesenteric
b. Median scaral
C. Renal arteries
D. Sup mesenteric

he superficial inguinal ring is an aperture in which structure?


A) Pubic tubercle
B) Aponeurosis external oblique
C) Aponeurosis internal oblique
D) Conjoint tendon
E) Transversus abdominus
A 32 year old woman undergoes an emergency caesarean section for failure to progress at 9 cm
cervical dilatation.which of the following correctly describes the pelvic shape which has an antero
posterior diameter of the inlet, greater than the transverse diameter?
A.Android
B.Anthropoid
C.Gynaecoid
D.Make
E.Platypelloid
Ans: (B) Anthropoid
The basic shapes of the pelvis are as follows:
• Gynaecoid pelvis (50%): normal female type, inlet is slightly transverse oval;sacrum is wide with
average concavity and inclination; subpubic angle is 90–100°.
• Anthropoid pelvis (25%): ape-like; anteroposterior (AP) diameters are long;transverse diameter
short; sacrum long and narrow, subpubic angle is narrow.
• Android pelvis (20%): male type, inlet is triangular or heart-shaped with anterior narrow apex,
subpubic angle is narrow < 90°.
• Platypelloid pelvis (5%): flat female type, AP diameter is short, transverse diameter is long,
subpubic angle is wide.

An 18 year old woman attends the gynaeology clinic complaining of urinary incontinence,3 months
after suffering a third degree perineal tear during a normal vaginal delivery
Which muscle forms the main bulk of the levator ani muscle?
A.Bulbocavernosus
B.Iliococcygeus
C.Ishiococcygeus
D.pubococcygeus
E.Urogenital diaphragm

Blood - testes barrier lie between


A) sertoli cells and sertoli cells
B) leydig cells and myoid cells
C) sertoli cells and germ cells
D) sertoli cells and spermatids

Rectum
A.Inferior rectal artery supplies rectum only
B .Inferior rectal nerve supplies rectum
C. Lined by simple columnar E with goblet cells
D.Measures 10.5 inches
E.Starts at the level of the sacroiliac joint as a continuation of sigmoid colon

Which artery provides the main blood supply of the breast ?


A. Axillary artery
B. Internal mammary artery
C. Lateral thoracic artery
D. Superior thoracic artery
E. Thoracoacromial artery
The lower third of the vagina has lymphatic drainage to which nodes?
Common iliac nodes
External iliac nodes
Internal iliac nodes
Superficial inguinal nodes
Para-aortic nodes

False about Ureter:


A.Starts at hilum of kidney.
B.Changes its direction at sacral promotory.
C.Penetrates bladder without any valve.
D.Enters bladder at lateral wall of trigone.
E.Enters pelvis in front of bifurcation of common iliac artery.

ANSWER : (B) .Changes its direction at sacral promotory.


-Ureter emerges from hilum of kidney and runs vertically downwards.
-Enters True/Lesser pelvis by crossing anterior to either bifurcation of common iliac artery infront of sacroiliac joint or at the
origin of external iliac vessels.
-Opposite the Ischial spines - changes direction - turns anteromediallyy to reach the base of bladder.
-Ureter opens into lateral angle of trigone of bladder.
-Passes obliquely / at an angle through the bladder wall.
-Thus intravesical oblique course has a valvular action but there is no true valve.

True about inferior mesentric artery:


A.Continues as inferior rectal artery
B.Takes part in formation of marginal artery
C.Arises from coeliac trunk
D.Supplies mid gut
E.Hepatic artery is a branch from it.
ANSWER : (B) Takes part in formation of marginal artery
-Inferior mesenteric artery arises from abdominal aorta.
-Supplies Hindgut.
-Continues as SUPERIOR RECTAL ARTERY.
-Takes part in the formation of Marginal Artery of Drummond.

Pain of ovarian pathology referred to:


A.Back of thigh.
B.Anterior Thigh
C.Medial Thigh
D.Gluteal region
E.Umblical region
ANSWER: (C) & (E) Medial Thigh &Umblical region
-Pain of ovarian pathology may b referred to Periumblical region (T10-T11) due to sensory fibres
accompanying sympathetics nerves to aortic plexus.
-Pain is also refereed along cutaneous distribution of Obturator nervee to medial aspect of thigh - as obturator
neurovascular bundle lies lateral to ovary in ovarian fossa.
Gartner's duct is present in:
A.Vaginal wall
B.Broad ligament of uterus.
C.Transverse cervical ligamnet
D.Perineal body
E.Utero vesical pouch.
ANSWER : (B) Broad ligament of uterus.
-Gartners duct is the duct of epophoron.
-Present in the broad ligament of uterus.
-Contents of broad ligament of uterus:
1.Fallopian tube.
2.Round ligament of uterus.
3.Uterine vessels.
4.Ovarian vessels in infundibulopelvic ligament.
5.Uterovaginal and ovarian nerve plexus.
6.Epophoron and its duct (Gartners duct).
7.Paroophoron
8.Some lymphnodes and lymphatics.
9.Dense connective tissue (Parametrium) on either side of uterus.

Where is the "Cave of Retzius" present?


A.Between urinary bladder and rectum.
B.Between urinary bladder and cervix
C.Infront of bladder
D.Between cervix and rectum.
ANSWER (C) Infront of bladder.
-Cave / Space of Retzius - Prevesical space.
- Extraperitoneal between pubic symphysis and bladder.
-Horse shoe shaped space.
-Intervenes between antero lateral pelvic wall and sides of bladder.

#ANATOMY #FACTS:
Arterial Supply of Vagina:
1.Vaginal branch of - Internal Iliac Artery -MAIN SUPPLY.
2.Cervicovaginal branch - Uterine Artery - UPPER PART.
3.Middle Rectal and Internal Pudendal arteries - LOWER PART

Which of the following is correct regarding embryological origion of the anal canal ?
A)Above pectinate line : derived endoderm ,superior rectal artery
B)Above pectinate line : derived endoderm,superior rectal artery
C)above pectinate line , derived ectoderm,columnar E pithellium
D)Below pectinate line :derived ectoderm,sup rectal artery
E)Below pectinate line , derived endoderm,middle and inf rectal artery

Which of the following runs outside of the ishiorectal fossa?


A)Pudendal canal
B)fat pad
C)inferior rectal nerve
D)Middle rectal nerve
E)perineal branch of s4 nerve

An 82 year old woman undergoes a vaginal hysterectomy for treatment of her procedentia. You are
revising the stages of the operation.
Which ligament runs laterally from the body of uterus, through the internal inguinal ring to the laium
majus ?
A)Broad ligament
B)Cardinal ligament
C) Iliolumber ligament
D)Round ligament
E)Uterosacral ligament

A 72 year old woman is reffered to the gyneocology outpatient clinic with a 2-day history of
postmenopausal bleeding . She subsequently undergoes a hysteroscopy and endometrial biopsy as
part of her investigation. Which of the following best describes the cells that line the uterus ?
A)Columnar epithelium
B)Cuboidal epithelium
C)pseudostartified columnar epithelium
D)Startified squamous E pithellium
E)Transitional cells

In the abdominal wall:


A The rectus abdominis muscle is attached to the crest of the pubis
B The posterior border of the external oblique muscle ends in the linea semilunaris
C The aponeurosis of the external oblique muscle takes part in the formation of the conjoint tendon
D The inferior epigastric artery is a branch of the internal iliac artery
E The conjoint tendon blends medially with the anterior layer of the rectus sheath
ANSWER : (A) The rectus abdominis muscle is attached to the crest of the pubis
-The rectus abdominis muscle originates from the symphysis pubis and pubic crest.
- It inserts into the 5th-7th costal cartilages and the xiphoid process.
-When the rectus abdominis is contracted its lateral margin forms a curved ridge called the linea semilunaris.
-The conjoint tendon is formed of the lower tendinous fibres of the internal oblique and transversus abdominis.
-Medially it attaches to the linea Alba, while it has a free border laterally.
-The conjoint tendon attaches to the posterior part of the rectus sheath and helps reinforce the posterior wall of
the inguinal canal.
-The inferior epigastric artery is a branch of the external iliac artery just above the inguinal ligament.

The obturator nerve:


A Emerges from the lateral border of psoas
B Is formed from the posterior divisions of the second, third and fourth lumber nerves
C Passes lateral to the internal iliac vessels
D Lies below the obturator artery in the obturator foramen
E Is separated from the normally sited ovary only be the pelvic peritoneum
ANSWER : (C) , (E) Passes lateral to the internal iliac vessels , Is separated from the normally sited ovary only
be the pelvic peritoneum
-It arises from the ventral divisions of L2-L3 L4
-Emerges on the medial border of the psoas at the pelvic brim, after descending through it.
-It passes behind the common iliac vessels and to the lateral side of the hypogastric vessels, which separate it
from the ureter.
-It enters the thigh through the upper part of the obturator foramen, after running above and in front of the
obturator vessels in the pelvis.
-The ovary lies in the ovarian fossa on the lateral wall of the pelvis, which is bound by the external iliac
vessels above, the obliterated umbilical artery in front, and the ureter behind.
-The obturator nerve crosses the floor of the fossa. The ovarian fossa is covered by peritoneum.

The external iliac artery:


A Enters the thigh anterior to the inguinal ligament
B At its origin is crossed by the ureter
C At its origin is crossed by uterine vessels
D Lies medial to the external iliac vein at its distal end
E Gives rise to the deep external pudendal artery
ANSWER : (B) At its origin is crossed by the ureter
-external ilaic artery enters the thigh by passing deep to the inguinal ligament to become the femoral artery.
-The common iliac artery is crossed by the ureter and ovarian vessels at its bifurcation.
-It lies lateral to the external iliac vein at its distal end.
-It gives rise to the inferior epigastric artery and the deep circumflex iliac branches.
-The external pudendal artery is a branch of the femoral artery

The obturator artery:


A Branches from the posterior trunk of the internal iliac artery
B Passes through the greater sciatic foramen
C Is crossed by the ureter
D Supplies the hip joint
E May be replaced by a branch of the superior epigastric artery
ANSWER : (D) Supplies the hip joint
-Branch of the anterior trun - Internal illiac
-Runs along the lateral wall of the pelvis with the obturator nerve.
-Pelvic cavity Relation : Obturator fascia - laterally
Ureter and peritoneum - medially,
Obturator nerve - Inferiorly.
-Leaves through the obturator canal via upper part of the foramen.
-Divides into medial and lateral branches
-Gives off muscular branches and an articular branch to the hip joint.
-The ureter crosses the bifurcation of the Common ilia Arteries and runs with the internal iliac arteries until the
ischial spines, where it turns to enter the bladder.
-The greater sciatic foramen contains:
1.Piriformis,
2.sciatic nerve,
3.post. Cut. Nerve of the thigh,
4.sup. & inf. gluteal nerves and vessels,
5.nerves to obturator internus & quadratus femoris,
6.pudendal nerve and internal pudendal vessels ("PIN" structures).
-The lesser sciatic foramen contains:
1. Obturator internus tendon,
2.Nerve to obturator internus,
3.Pudendal nerve,
4.internal pudendal artery and vein.
5.An aberrant obturator artery may be found branching from the inferior epigastric artery or external iliac
artery.

The internal pudendal artery:


A Leaves the pelvis through the lesser sciatic foramen
B Lies on the medial wall of the ischial rectal fossa
C Has a branch which pierces the perineal membrane
D Gives rise to the middle rectal artery
E Supplies the upper vagina
**************
ANSWER (C) Has a branch which pierces the perineal membrane
-The artery is a branch of the anterior trunk of the internal iliac Artery.
-Leaves the pelvis through the greater sciatic foramen
-Enters the perineum through the lesser sciatic foramen.
- It runs along the lateral wall of the ischiorectal fossa
-Passes forward between the two layers of the urogenital diaphragm (the perineal membrane is the inferior
fascial layer)
-Pierces the inferior fascia.
-It gives off the inferior rectal artery and branches to the labia and clitoris (penis).
-The middle rectal artery is a branch of the anterior division of the internal iliac artery.
-The vaginal arterial supply is from the vaginal artery (internal iliac) and superior vesicular artery.

Which one of the above structures lie anterior to the pelvic sympathetic trunk
A.rectum
B.the sacrum
C. descending aorta
D.inferior vena cava
E.internal iliac vessels
Key is A
Explanation: pelvic sympathetic trunk
Continuous with the abdominal part behind the common iliac vessel
Lies posterior to rectum
Lies anterior to sacrum
Lies medial to the anterior sacral foraminia
Has 4-5 segmentally arranged ganglia
Which one of the above organs is correctly paired with it's parasympathetic nerve supply
A.urinary bladder - lumber splanchnic nerve
B.small intestine - pelvic splanchnic nerve
C.descending colon - vagus nerve
D. Vagina - lumbar splanchnic nerve
E. Ovary- vagus nerve

The nerve supply to the body of uterus


A. Aortic plexus
B. Superior hypogastric plexus
C. Inferior hypogastric plexus
D. Superior mesenteric plexus
E. Inferior mesenteric plexus

The blood supply of Fallopian tube


A. Ovarian artery
B.uterine artery
C. Ovarian and uterine artery
D.external iliac artery
E.internal iliac artery

Venous drainage of right ovary


A. Superior mesenteric vein
B. Inferior vena cava
C. Left renal vein
D. Right renal vein
E. inferior mesenteric vein

Correct relation of round ligament with its embryological origin and anatomy is:
A-Gubernaculum ovarii. In front of the uterine cornu to the labia majora
B-Gubernaculum ovaria. In front of the uterine cornu to the ovarian ligament
C-Paramesonephric duct. In front of the uterine cornu to the labia majora
D-Paramesonephric duct. Behind the uterine cornu to the labia majora

Which three muscles form the superficial group of muscles of the perineum?
A Bulbospongiosus, superficial transverse perineal muscle, and ischiocavernous
B Deep transverse perineal muscle, external anal sphincter, and bulbospongiosus
C External vaginal sphincter, deep transverse perineal muscle, and external anal sphincter
D Iliococceageus, ischiocavernosus, and bulbospongiosus
E Levator ani, ischiocavernous, and bulbospongiosus

10.A baby delivered after shoulder dystocia was diagnosed to have Erb’s palsy. Which
component of the brachial plexus is most likely to be involved?
A C5
B C5, C6
C C7, C8
D C7, C8, T1
E C8, T1

The distal transverse colon is supplied by which artery?


a- Superior mesenteric artery
b- Inferior mesenteric artery
c- Superior rectal artery
d- Middle colic artery
e- Inferior pancreatoduodenal artery

Which of the following is the innermost layer of the testis?


a- Cremasteric fascia
b- Tunica vasculosa
c- Tunica albuginea
d- Tunica vaginalis visceral
e- Tunica vaginalis parietal

Which of the following is contained within the deep perineal pouch?


a- Proximal portion of urethra
b- Fatty layer superficial fascia
c- Ischiocavernosus muscle
d- Bulbospongiosus muscle
e- Superficial transverse perineal muscle

The following statement about the adrenal glands is correct:


A They lie posterior to the diaphragm
B The left adrenal gland lies behind the pancreas
C Lymphatic drainage is to the superficial inguinal nodes
D The adrenal cortex contains chromaffin cells
E The adrenal medulla is derived from mesoderm
****************
ANSWER:(B) The left adrenal gland lies behind the pancreas
-The adrenal glands rest posteriorly - on the diaphragm which implies they are anterior to diaphragm.
-Left adrenal lies behind the pancreas.
-The lymph vessels pass to the lateral aortic nodes.
-The adrenal medulla makes catecholamines and contains chromaffin cells.
-It is derived from neural crest cells (ectoderm), while the cortex is derived from mesoderm.

Which of the following is a branch of the posterior division of the internal iliac artery?
A Inferior gluteal artery
B Internal pudendal artery
C Obturator artery
D Superior gluteal artery
E Uterine artery

Which of the following describes the anatomy of the inguinal region?


A The deep inguinal ring lies at the lateral two-thirds of the inguinal ligament
B The deep inguinal ring transmits the ilioinguinal nerve
C The superficial inguinal ring lies below the pubic tubercle
D The superficial inguinal ring transmits the genitofemoral nerve
E The superficial inguinal ring transmits the round ligament
*****************************************
Answer: ( E ) The superficial inguinal ring transmits the
round ligament
-The deep inguinal ring is situated at the midpoint of the inguinal ligament.
-It can be located by finding the midpoint between the anterior superior iliac spine and the pubic
tubercle.
-The superficial inguinal ring lies just above, and lateral to the
pubic tubercle.
-The deep and superficial rings mark the entrance (deep ring) and exit (superficial ring) to the
inguinal canal.
-The canal’s boundaries are:
• Anterior wall: external oblique aponeurosis, with lateral reinforcement from the
internal oblique
• Posterior wall: transversalis fascia, with the conjoint tendon (internal oblique and
transversus abdominis) providing medially
• Superiorly: internal oblique
• Inferiorly: inguinal ligament
-Running through the canal is the round ligament in females and the spermatic cord in males.
-The ilioinguinal nerve passes through the superficial inguinal ring only,having travelled down the
lateral abdominal wall between the internal and externaloblique muscles.

A 26-year-old woman attends her general practitioner for her 6 week postnatal
check. She has a central abdominal protrusion which is diagnosed as a divarication
of the rectus muscle.
What is the nerve supply to the rectus abdominis muscle?
A T2–T12
B T7–T12
C T12–L3
D L2–L5
E L5–S3

All true for The gynaecoid pelvis , except:---


A. is the commonest type of female pelvis
B. has a wider aneroposterior than transvers diameter at the brim
C. has a shallow , wide sacrosciatic notch
D. has parallel pelvic side – walls

A 21-y/o marine biologist asks about her first bimanual examination and it is explained to her that the
normal position of the uterus is:
A.Anteflexed and anteverted
B.Retroflexed and anteverted
C.Anteflexed and retroverted
D.Retroverted arid retroflexed
E.Anteverted and retroverted

An obstetrician performs a median episiotomy on a woman before parturition to prevent uncontrolled


tearing. If the perineal body is damaged, the function of which of the following muscles might be
impaired?
A.Ischiocavernosus and sphincter urethrae
B.Deep transverse perineal and obturator internus
C.Bulbospongiosus and superficial transverse perineal
D.External anal sphincter and sphincter urethrae
E.Bulbospongiosus and ischiocavernosus
A 42-year-old woman who has had six children develops a weakness of the urogenital diaphragm.
Paralysis of which of the following muscles would cause such a symptom?
A.Sphincter urethrae
B.Coccygeus
C.Superficial transversus perinei
D.Levator ani
E.Obturator internus
he key is A, the urogenital diaphragm consists of the sphincter urethrae and deep transverse perineal muscles. Weakness of the
muscles, ligaments and fascia of the pelvix floor such as the pelvic diaphragm and cardinal (transverse cervical) ligament oc curs
result of multple child delivery, advancing age and menop ause. The pelvic diaphragm is composed of levator ani and coccygeus
muscles. The superficial transverses perinei is one of the superficial perineal muscles and the obturator internus forms the lateral
wall of the ischiorectal fossa

A 37-year-old man is suffering from carcinoma of the skin of the glans penis. Cancer cells are likely
to metastasize directly to which of the following lymph nodes?
A.External iliac nodes
B.Internal iliac nodes
C.Superficial inguinal nodes
D.Aortic (lumbar) nodes
E.Common iliac nodes

A 29-year-old woman with a ruptured ectopic pregnancy is admitted to a hospital for culdocentesis.
A long needle on the syringe is most efficiently inserted through which of the following structures?
A.Anterior fornix of the vagina
B.Posterior fornix of the vagina
C.Anterior wall of the rectum
D.Posterior wall of the uterine body
E.Posterior wall of the bladder

A 58-year-old man is diagnosed as having a slowly growing tumor in the deep perineal space. Which
of the following structures would most likely be injured?
A.Bulbourethral glands
B.Crus of penis
C.Bulb of vestibule
D.Spongy urethra
E.Great vestibular gland
Key is A .... the deep perineal space contains the bulbourethral (cowper's) glands. The crus of the penis, bulb of the
vestibule, spongy urethra and great vestibular gland are found in the superficial perineal space

A 46-year-old woman has a history of infection in her perineal region. A comprehensive examination
reveals a tear of the superior boundary of the superficial perineal space. Which of the following
structures would most likely be injured?
A.Pelvic diaphragm
B.Colles' fascia
C.Superficial perineal fascia
D.Deep perineal fascia
E.Perineal membrane

A young couple is having difficulty conceiving a child. Their physician at a reproduction and fertility
clinic explains to them that:
A.The ovary lies within the broad ligament
B.The glans clitoris is formed from the corpus spongiosum
C.Erection of the penis is a sympathetic response
D.Ejaculation follows parasympathetic stimulation
E.Fertilization occurs in the infundibulum or ampulla of the uterine tube.

A 37-year-old small business manager receives a gunshot wound in the pelvic cavity, resulting in a
lesion of the sacral splanchnic nerves. Which of the following nerve fibers would primarily be
damaged?
A.Postganglionic parasympathetic fibers
B.Postganglionic sympathetic fibers
C.Preganglionic sympathetic fibers
D.Preganglionic parasympathetic fibers
E.Postganglionic sympathetic and parasympathetic fibers

A 29-year-old woman is admitted to a hospital because the birth of her child is several days overdue.
Tearing of the pelvic diaphragm during childbirth leads to paralysis of which of the following
muscles?
A.Piriformis
B.Sphincter urethrae
C.Obturator internus
D.Levator ani
E.Sphincter ani externus

A 59-year-old woman comes to a local hospital for uterine cancer surgery. As the uterine artery
passes from the internal iliac artery to the uterus, it crosses superior to which of the following
structures that is sometimes mistakenly ligated during such surgery?
A.Ovarian artery
B.Ovarian ligament
C.Uterine tube
D.Ureter
E.Round ligament of the uterus

A 22-year-old woman receives a deep cut in the inguinal canal 1 inch lateral to the pubic tubercle.
Which of the following ligaments is lacerated within the inguinal canal?
A.Suspensory ligament of the ovary
B.Ovarian ligament
C.Mesosalpinx
D.Round ligament of the uterus
E.Rectouterine ligament

On a busy Saturday night in Chicago, a 16- year-old boy presents to the emergency room with a
stab wound from a knife that enters the pelvis above the piriformis muscle. Which of the following
structures is most likely to be damaged?
A.Sciatic nerve
B.Internal pudendal artery
C.Superior gluteal nerve
D.Inferior gluteal artery
E.Posterior femoral cutaneous nerve
key is C, the superior gluteal nerve leave the pelvis through the greater sciatic foramen above the piriformis. the
sciatic nerve, internal pudendal vessels, inferio r gluteal vessels and nerve and the posterior femoral cutaneous nerve
leave the pelvix below the piriformis

A 53-year-old bank teller is admitted to a local hospital for surgical removal of a benign pelvic tumor
confined within the broad ligament. There is a risk of injuring which of the following structures that
lies in this ligament?
A.Ovary
B.Proximal part of the pelvic ureter
C.Terminal part of the round ligament of the uterus
D.Uterine tube
E.Suspensory ligament of the ovary

A 32-year-old patient with multiple fractures of the pelvis has no cutaneous sensation in the
urogenital triangle. The function of which of the following nerves is most likely to be spared?
A.Ilioinguinal nerve
B.Iliohypogastric nerve
C.Posterior cutaneous nerve of the thigh
D.Pudendal nerve
E.Genitofemoral nerve
Key is B....The iliohypogastric nerve innervates the skin above the pubis. The skin of the urogenital triangle is
innervated by the pudendal nerve, perineal branch of posterior femoral cutaneous nerve, anterior scrotal/ labial
branches of ilioinguinal nerve, and hte genital branch of the genitofemoral nerve

To perform an elective lower segment caesarean section, the obstetrician


makes a transverse suprapubic incision. Which of the following abdominal wall
layers will not be encountered transected during this incision?
A. Anterior rectus sheath
B. Posterior rectus sheath
C. Rectus abdominis muscle
D. Skin and subcutaneous tissue
E. Transversalis fascia and parietal peritoneum

Deep perineal pouch contains


A) Bulb of Penis
B) crura of Penis
C) bulbospongiosus
D) membranous urethra

Which muscle forms part of the inguinal liagament ?


A)External oblique
B )Internal oblique
C)Internal oblique
D) Rectus abdominis
E)Transversus abdominis

Which muscle enters the abdomen behind the medial arcuate ligament ?
A)External oblique
B)Illiacus
C)psoas
D)Pyramidalis
E)Transversus abdominis

Which muscle lies within the rectus sheath and is supplied by the sub costal nerve ?
A )External oblique
B) Internal oblique
C)Pyramidalis
D)Rectus abdominis
E)Transversus abdominis
this structure enters inguinal canal and is identified easily at laparoscopy :
a. Inferior epigastric artery
b. Superior epigastric artery
c. Round ligament
D. None of above

the plane of pelvic inlet as at what angel of horizontal?


a.15
b.30
c.45
d.60
e.75

which of the following is true of great saphenous vein?


a. Ascends posterior to the medial malleolus.
b.passes through colles fascia.
c.passes through femoral canal.
d.receive blood from the post tibial veins.
e.Receivess the superficial external pudental vein.

Which of the following statements regarding the duodenum is true?


The blood supply to the duodenum is from the superior mesenteric artery
The duodenum commences at the ileocaecal opening
The duodenum has an extensive mesentery
The duodenum is a retroperitoneal structure
The duodenum is approximately 5 meters long

Regarding the structure of the detrusor muscle. Which of the following is true?
A) The detrusor is a single layer of longitudinal smooth muscle
B) The detrusor is a single layer of circular smooth muscle
C) The detrusor is a single layer consisting of both circular and longitudinal muscle fibres
D) The detrusor is divided into 2 layers consisting of an inner longitudinal smooth muscle
layer and outer circular smooth muscle layer
E) The detrusor is divided into 3 layers consisting of inner and outer layers of longitudinal
smooth muscle with a middle circular smooth muscle layer

Which two nerves provide the primary cutaneous sensory innervation to the labia majora?
A) Ilioinguinal and inferior rectal
B) Ilioinguinal and pudendal
C) Pudendal and perineal
D) Anterior femoral and genitofemoral
E) Pudendal and iliohypogastric

Which of the following statements regarding the Vaginal artery is typically TRUE?
a. It arises from the Internal iliac artery
b.It arises from the External iliac artery
c. It arises from the Abdominal Aorta
d. It arises from the Ovarian artery
e.It arises from the Uterine artery

The testis receive innervation from which spinal segment


a.T10
b.T12
c.S1
d.S2

The anterior scrotal artery is a branch of which artery?


a.Internal Pudendal
b.External Pudendal
c.Cremasteric
d.Testicular
e.Gonadal
Ant > external
Post > internal

25. Which of the following molecules generates weak forces that can attract neutrophils to cell walls?
Cadhedins
Eicosanoids
Hemidesmosomes
Integrins
Selectins

23. Which part of the cell cycle is noted for sister chromatids separating and moving to opposite
sides of the cell?
Anaphase
Metaphase
Prophase
Synthesis phase
Telophase
Mitosis starts with chromosomes condensing (prophase)
follow by chromatids aligning (metaphase),
following by sister chromatids separating and moving (anaphase). Finally the cell membrane divides in telophase.

20. What is the lymphatic drainage of the ovaries?


Common iliac nodes
External iliac and superficial iliac nodes via the round ligament
External iliac nodes
Internal iliac nodes
Lateral aortic and preaortic nodes

16. Which artery is a direct branch of the aorta?


Inferior vesical
Internal iliac
Ovarian
Uterian
Vaginal

19. A 30-year-old woman has developed haematuria, pain and reduced urine output on day 3
following a total abdominal hysterectomy. A urine dipstick showed red blood cells but no nitrites. An
X-ray of the pelvis was unremarkable. What is the most likely diagnosis?
Bladder injury
Intra-abdominal bleeding
Ureteric stone
Ureteric trauma
Urinary tract infection

18. You have been asked to suture a deep episiotomy by a midwife. On examination you notice that
the external anal sphincter is mostly torn, with a few fibres left intact. The internal anal sphincter is
intact. How would this tear be described?
Second-degree tear
Third-degree tear, 3a
Third-degree tear, 3b
Third-degree tear, 3c
Fourth-degree tear

17. A 65-year-old woman underwent vaginal surgery for which she was in the lithotomy position for 2
½ hours. Postoperatively you have noticed parasthesia of the lateral side of the leg, foot and foot
drop. The compression of which nerve is likely to be responsible?
Common peroneal nerve
Femoral nerve
Great saphenous nerve
Obturator nerve
Pudendal nerve

15. A 28-year-old woman presents to the gynaecology clinic with a history of dysmenorrhoea,
dysparunia and menorrhagia. A vaginal examination reveals a thickened nodule in the posterior
fornix. Which anatomical structure is most likely to be affected?
Constipated bowel
Ovaries
Pectineal ligaments
Ureter
Uterosacral ligaments

14. What is the most serious risk associated with the insertion of a Veress needle through the
umbilicus in a thin patient, an outcome that is unlikely with the Hasan entry technique?
Bowel perforation
Damage to liver
Damage to ureters
Inferior epigastric artery injury
Vascular injury
Key is E
The answer is vascular injury. The aorta, inferior vena cava and common iliac arteries are put at risk with the insertion
of the Veress needle. The open entry technique is considered to be safer in this matter. The risk of bowel damage is
not completely avoided using the open entry technique .
13. Which structure is lined by non-keratinised stratified sq epi?
Endocervix
Fallopian tubes
Skin
Ureters
Vagina

12. Ischemic injury to the pelvic ureter may be caused by which one of the following arteries?
Aorta
Common iliac artery
Inferior vesical artery
Renal artery
Uterine artery

11. A 20-year-old woman presents with a painful lump in the vagina. The examination confirms
Bartholin’s abscess. Which one of the following glands is involved?
Bulbo urethral glands (Cowper’s)
Glands of skini
Greater vestibular gland
Lesser vestibular gland
Seminal vescicle

8. Sympathetic supply to the bladder is derived from which lumbar segments?


L1 and L2
L2 and L3
L3 and L4
L4 and L5
S2 , S3 and S4
Key is A
The answer is L1 and L2. Nerve root supply for pelvic organs is a common single best answer examination question.
It is worth drawing a table for each of the main pelvic organs and writing down the artery, venous, lymphatic and
nerve supply. Note that the parasympathetic supply for the bladder comes from S2, S3 and S4 but the sympathetic is
as above. Sensory nerves are found in both supplies

The uterine vein drains where?


a.Internal iliac vein
b.Splenic vein
c.Renal vein
d.Inferior vena cava
e.Hepatic portal vein

Which part of the nephron is responsible for 100% of the glucose reabsorption in a healthy adult
kidney?
a.Bowmans capsule
b.Ascending loop of Henle
c.Descending loopof Henle
d.Proximal convoluted tubule
e.Distal convoluted tubule

The medial umbilical ligament is a remnant of what structure?


a.Fetal paraumbilical veins
b.Fetal umbilical arteries
c.Lateral umbilical fold
d.Ligamentum teres
e.Urachus

The pelvic outlet is bounded posteriorly by what?


a.sacral prominence
b.anterior border piriformis
c.posterior border piriformis
d.Tip of the coccyx
e.Ischial tuberosities

Which cell type of the testis secrete inhibin?


a.Epithelial cells
b.Germ cells
c.Sertoli cells
d.Leydig cells

A patient attends clinic with a vaginal prolapse. On examination the vaginal prolapse is visible 1.5cm
above the plane of the hymen. According to the POPQ classification what
grade is this prolapse?
a.Grade 0
b.Grade 1
c.Grade 2
d.Grade 3
e.Grade 4

What is the anterior boundary of the pelvic outlet?


a.pubic arch
b.ischial tuberosity
c.pubic crest
d.pubic tubercle
e.pubic ramus

Sensory supply to the clitoris is via branches of which nerve?


a.Ilioinguinal
b.Perineal
c.Superior rectal
d.Inferior rectal
ePudendal nerve

Which of the following nerves passes through the inguinal canal?


a.Ilioinguinal nerve
b.Pudendal nerves
c. Lateral femoral cutaneous nerve
d.Inferior hypogastric
e.Pelvic splanchnic nerves

The sacroiliac joint is what type of joint?


a.Condyloid
b. Synovial
c. Primary Cartilaginous
d. Secondary Cartilaginous
e. Fibrous
You are called to see a women after a prolonged labour with failed instrumental delivery converted
to c-section. She complains of pins and needles to the left thigh. On examination the anterior left
thigh including the anterior knee has reduced sensation and there is reduced power on testing knee
extension (MRC grade 4 left vs 5 right). What is the likely diagnosis?
a.Common peroneal palsy
b.Obturator palsy
c.Genitofemoral nerve palsy
d. Femoral nerve palsy
e.Left L2 nerve palsy

Regarding the female breast how many lobules does the typical breast contain?
a.1
b. 4
c.5-9
d.15-20
e.40-50

Which artery is associated with the midgut?


a. Coeliac Trunk
b. External Iliac
c. Internal Iliac
d. Inferior Mesenteric
e.Superior Mesenteric

Which of the following takes part in the arterial supply of the ovary?
a.Uterine arteries
b. Vaginal arteries
c. Renal arteries
d.Ureteric arteries
e.None of the above

31.Which of the following is correct concerning the umbilical artery?


Is a branch of the external iliac artery
Is normally a single structure
Passes through the liver
Returns deoxygenated blood to the placenta in utero

11.Which of the following is correct regarding the inguinal canal?


a.Commences at the superficial ring which is a defect in the fascia transversalis
b.Extends from the deep inguinal ring downward and laterally to the superficial inguinal ring Incorrect
answer selected
c.Is approximately 7 cm long in the adult
d.Lies below the inguinal ligamene.
e.Transmits the ilioinguinal nerve

10.Which of the following is true regarding the inferior hypogastric (pelvic) plexus?
a.Contains sensory fibres
b.Extends into the round ligament of the uterus Incorrect answer selected
c.Gives rise to the vesical plexus
d.Is situated on the side of the anal canal
e.Lies lateral to the internal iliac vessels
7. Which muscle of the lower limb is primarily responsible for eversion of the foot?
a.Extensor hallucis longus
b.Iliopsoas
c.Peroneus longus
d.Quadriceps

5.Which muscle of the lower limb is primarily responsible for ankle dorsiflexion?
(Please select 1 option)
a..Extensor hallucis longus
b.Iliopsoas
c.Peroneus longus
d.Quadriceps
e.Tibialis anterior

17‐ year‐ old girl presents to you six weeks pregnant.


After discussion with her family and yourself she has decided that she would like a termination. She
is terrified of having an operation and requests a medical termination. You explain the process and
give her mifepristone. What is the action of this drug?
A Imitates GnRH
B Imitates oxytocin
C Inhibits HCG
D Inhibits the action of oestrogen
E Inhibits the action of progesterone

Which of the following nerve roots, if compressed by a herniated disc, would result in a diminished
knee jerk?
A L2
B L4
C L5
D S1
E S2

Which of the following is not a branch of the pudendal nerve?


A Perineal nerve
B Dorsal nerve of the penis
C Inferior rectal nerve
D Genitofemoral nerve
E Posterior scrotal nerve

Which vessel is commonly injured during sacrospinous ligament fixation??


A. Inferior gluteal artery
B. Internal pudendal artery
C. Inferior rectal artery
D. Coccygeal branch of gluteal artery

The following structures are located at the transpyloric plane


A.Hilum of left kidney
B.Filum terminale
C.Neck of the pancreas
D.Origin of the inferior mesentric artery
E.10th costal cartilage
Diaphragm
A.Vena caval opening located at the level of T10
B.Aortic opening transmits vagus nerve
C.oesophageal opening encircled by the left crus
D.Medial arcuate ligament transmits quadratous lumborum
E.Phrenic nerve supply the diaphragm with sensory and motor fibers with a root value from C3,4 and
5

Which muscle of the lower limb is primarily responsible for knee extension?
A Extensor hallucis longus
B Iliopsoas
C Peroneus longus
D Quadriceps
E Tibialis anterior

Which muscle of the lower limb is primarily responsible for hip flexion?
A Extensor hallucis longus
B Iliopsoas
C Peroneus longus
D Quadriceps
E Tibialis anterior

Which one of the following branches of the abdominal aorta gives rise to the left gastric, splenic and
hepatic arteries?
A.Adrenal arteries
B.Coeliac trunk
C. Common illiac arteries
D.Gonadal ( ovarian or testicular) arteries
E.Inferior mesentric arteries

Which one of the following dermatomal levels innervates the fifth digit of the foot?
A.L4
B.L5
C.S1
D.S2
E.S3

Lumbar plexus
A.Formed within the substance of the quadratous lumborum muscle
B.Femoral nerve arises from posterior division of the L2,3 and 4 roots
C.Ilioingunal illiohypogastric nerves arises from the anterior division of L1 and 2
D. Give rise to posterior femoral cutaneous nerve
E.Obturator nerve is purely muscular

Long saphenous vein


A.Begins as a continuation of the media end of dorsal venous arch in front of the medial malleolus
B.Runs Anterior to the saphaneois nerve in the leg
C.Has more valves in the thigh compared to the leg
D. Pierce cribriform fascia to join the femoral vein 4 cm inferolateral to the symphysis pubis
E.Does not communicate with the deep venous system except when it join the femoral vein

Which of the following is true regarding pudendal nerve?


A) arises frim S3,4,5
B) enters the perineum through obturator canal
C) leaves the pelvis through greater sciatic foramen
D) lies medial to the ischial spine
E) lies on the supraspinatus ligament

Which of the following is correct regarding the levator ani muscle


A) are contracted during defecation
B) are innervated from S2,3,4
C) are relaxed during micturation
D) are involuntary muscles
E) form a floor which slopes downwards, laterally and backwards

A 47 - year old woman under goes a routine trabsabdominal hysterectomy to remove a large fibroid
uterus. She is found to have a fibroid in the broad ligament and there is concern that her ureter may
have been damaged due to the difficult operation. With regards to the path of the ureter, which of the
following is correct?
A.In the broad ligament, both ureters pass over their respective uterine artery
B.Runs laterall to the internal illiac artery
C.Ovarian vessels enter the pelvic posterior to the ureters
D.Upper one third of the ureters lie in the abdomen
E.Ureters cross close to the bifurcation of the common illiac vessels

Which of the following organs is derived from ecto dermal neural crest cells ?
A.Adrenal gland inner medulla
B.Adrenal gland outer cortex
C.Liver
D. Pancreas
E. Spleen

A 21 year old woman under goes a laproscopic ovarian cystectomy to remove a dermoid cyst .three
days after the operation she presents to the emergency department feeling un well and her
haemoglobin level is found to be 6 g/dl . Damage to a blood vessel is suspected from the
laparoscopic procedure
Which vessel cross the common and external illiac artery in the infudibulopelvic fold?
A.Femoral artery
B.Inferior mesentric artery
C.Median sacral artery
D.Ovarian artery
E.Renal artery

Adductor canal
A.Lies deep to the adductor Magnus muscle
B.Contains femoral nerve
C.Lower part of adductor longus spilits to form the adductor hiatus
D.Adductor hiatus transmit the saphenous nerve
E.Femoral vein and artery leave the canal to reach the popliteal fossa via the adductor hiatus

Regarding Femoral triangle


A.Lateral border is formed by the lateral border of sartorius
B.Contains femoral artery, vein and nerve enclosed into the femoral sheath
C. Contains superficial inguinal lymph note in the femoral canal
D.Medial border is formed by the medial border of adductor longus
E. Femoral ring is wider in males if compared to females
Inguinal canal
A.Deep ring lies medial to the inf epigastric artery
B.Posterior wall is formed by the internal oblique muscle
C.Deep ring is located just above the mid point of inguinal ligament
D.Floor is formed by the conjoined tenon
E. Contains the genital branch of genitofemoral nerve through its length

Rectus sheath
A.Is a complete envelope formed by the abdominal muscles aponeurosis
B.Contains superior and inferior epigastric arteries lower 5 intercostal and subcostal nerves
C. Posterior walk is deficiency below the level of umblicus
D.Anterior wall is formed by the aponeurosis of the 3 abdominal muscles at the level of umblicus
E. Pyramidalis lies superficial to the anterior wall just above the symphysis pubis

How many seminiferous tubules would you typically expect to find in a testicular lobule?
A2
B20
C 200
D2000
E2 million

A 27 year old woman has a cervical smear result which shows border line changes, which cells line
the ecto cervix?
A.Cilliated cells
B.Columnar epithelium
C.Cuboidal epithelium
D.Smooth muscle cells
E.startified squamous epithelium

Which vessel provides blood supply to the intestine from the splenic flexure of the transverse colon
to the rectum
A.Inferior mesentric artery
B.Median sacral artery
C.Middle comic artery
D.Rectal array
E.Superior mesentric artery

Which of the following structures does not pass through the diaphragm?
A.Azygos vein
B.Cisterna chyli
C.Inferior vena cava
D.Oesophagus
E.Thoracic duct

Rectus Abdominis
A.Has 3 tensinous intersections attached to the anterior rectus sheath
B.Originates from the xiphoid process
C.Inserted in the pubic crest and tubercle
D.Lateral border is know as lines alba
E.All of the above
Anterior abdominal wall
A.External oblique muscle originates from lower 8 costal cartilages
B.Transversus abdominis muscle originates from linea alba
C.Conjoined tendon is formed by the lower most fibers of external oblique muscle
D.Internal oblique muscle originates from the lateral 2/3 of inguinal ligament, pubic crest.,illiac
crest,and thoracolumber fascia
E.Half inguinal is attached to the posterior wall of rectus sheath

Anal canal
A.lower 2/3 of the mucosa is endodermal in origin
B.Internal anal sphincter is integrated by inferior rectal nerve
C. Pectinate line is located above the Hilton (white line)
D.Superficial part of the external anal sphincter has no boney attachment
E.Middle 1/3 is drained by systemic veins

Concerning the inguinal canal.


a. .it transmits the ilioinguinal nerve
b. .the deep inguinal ring lies below the midpoint of the inguinal ligament
c. .the superficial inguinal ring overlies the pubic tubercle
d. .medially the anterior wall is made up of the external oblique apon
e. .laterally the posterior wall is formed by the conjoint tendon

The vagina.
a. .has an anterior wall longer than the posterior wall
b. .contains mucus secreting glands in its epithelium
c. .is related in its lower third to the bladder base
d. .during reproductive life has an acid pH
e. .is derived from the mesonephric

The mature ovary.


a. .is attached to the anterior aspect of the broad ligament by the mesovarium
b. .is covered by a thick capsule
c. .has a surface layer of germinal epithelium
d. .on the right side drains venous blood to the renal vein
e. .has lymphatics which pass to the external iliac nodes

The extradural(epidural) space.


a. .contains the internal vertebral plexus
b. .lies between the pia matter and dura matter
c. .contains no cerebrospinal fluid
d. .ends at the level of the second lumbar vertebra
e. .does not extend laterally through the intervertebral foramina

The obturator nerve.


a. .is derived in pat from the second lumbar nerve
b. .supplies both hip and knee joints
c. .runs superficial to the common iliac vessels
d. .leaves he pelvis through the greater sciatic foramen
e. .supplies the abductor muscles of the thigh

The right ovarian artery.


a. .arises from the abdominal aorta above the renal artery
b. .passes posterior to the third part of the duodenum
c. .passes posterior to the genitofemoral nerve
d. .supplies the right ureter
e. .anastomoses with the right uterine artery

Wrong statement among following is


A.Cervix projects into anterior vaginal wall
B.posterior vaginal wall is 10 cm
C.Anterior to vagina is pouch of Doughlas
D.posterior to vagina is rectum and anal canal
E.Laterally ureter and levator ani

Female urethra
A.Drains lymph directly to internal illiac and indirectly to external illiac lymph nodes
B.Measures 4 cm in children
C.External uretheral sphincter is located around the lower 1/3
D. Receives the opening of the parauretheral skenes glands near its distal part
E.Lined by transitional epithelium throughout its length

Female urinary bladder


A.Musculosa is formed by the dartos muscle
B.Obliterated urachus forms the medial umblical ligament
C.Trigone is endodermal in o rigion
D.Sympathetic innervation is derived fromL1,2,3 roots
E. Fundus lymphatic trains to the external illiac lymph nodes

Female ureter
A.Intra abdominal part is longer than the pelvic part
B.Crossed by the uterine artery 2 cm lateral to the cervix
C.Is 50 cm in length
D. Right ureter is more dilated than the left
E. Joins the base of the bladder lateral to the vaginal wall

Regarding ovarian anatomy


A.Germinal epithelium lies under the tunics albuginea
B.Infudibulo pelvic ligament connect the ovary to the posterior layer of broad ligament
C.Anterior boundary of the ovarian fossa is formed by the external illiac vessels
D.Ovarian ligament is attached to the lower(medial) pole
E.Has no parasympathetic innervation

Which of the following muscles is contained within the superficial perineal pouch
A External sphincter muscle of urethra
B Deep transverse perineal muscle
C Compressor urethrae muscle
D Urethrovaginal sphincter
E Ischiocavernosus muscle

The inguinal canal is reinforced posteriorly by which structure?


A Conjoint tendon
B Inguinal ligament
C External oblique aponeurosis
D Internal oblique fibres
E Lacunar ligament
Regarding Levator Ani which of the following statements is TRUE?
A Arterial blood supply is from inferior rectal artery
B Arterial blood supply is from superior rectal artery
C Arterial blood supply is from internal iliac artery
D Arterial blood supply is from inferior gluteal artery
E Arterial blood supply is from superior gluteal artery

Fallopian tube
A.measures 15 cm
B.Infundibulum is related to the medial border of the ovary
C.Has a 3 muscle coats with inner circular and outer longitudinal
D.It does not show cyclical changes during menstrual cycle
E.Fertilization occurs in the infundibular part

Pelvic peritoneum
A.Anterior supravaginal Cervix is not covered by peritoneum
B.Base of the bladder is covered by peritoneum
C.Ovaries are partially covered by peritoneum
D.Rectum is completely covered by peritoneum
E.Anal canal is partially covered by peritoneum

Medial part of labia majora is supplied by


A.Ilioinguinal nerve
B.Genitofemoral nerve
C.Pudendal nerve
D. Iliohypogastric nerve
E.Obturator nerve

Greater vestibular glands open at


A.posterior part of cervix
B.lateral wall of vagina at introitus
C. Groove between vaginal orrifice and posterior part of labia minora
D.Anterior part of vagina
E.Near uretheral opening

Concerning the abdominal wall.


a. .the umbilicus is located in the territory of the L1 dermatome
b. .the rectus abdominis muscle has attachments to the anterior wall of the rectus sheath
c. .the left and right epigastric arteries anastomose
d. .distended veins radiating from the umbilicus are indicative of portal hypertension
e. .Langer's lines run vertically over the lower abdomen

Concerning the thigh.


a. .rectus femoris forms the lateral boundary of the femoral triangle
b. .the femoral nerve enters the thigh within the femoral sheath
c. .the ilioinguinal nerve innervates skin over its medial aspect
d. .both the saphenous nerve and the femoral artery pass through the adductor canal
e. .the lacunar ligament is the medial border of the femoral ring

The pelvic surface of the sacrum.


a. .gives origin to the piriformis muscle
b. .gives origin to the levator ani muscle
c. .is broader in the male than in the female
d. .transmits the dorsal rami of sacral nerves
e. .is in contact with the anal canal

The pudendal nerve.


a. .arises from the posterior rami of S2,3 and 4
b. .leaves the pelvis through the lesser sciatic foramen
c. .crosses the ischial spine on the lateral side of the internal pudendal artery
d. .supplies the levator ani
e. .supplies the clitoris

The following statements about the diaphragm are correct.


a. .the greater sciatic nerves pierce the central tendon
b. .the aortic aperture transmits the thoracic duct
c. .the vena caval aperture transmits the left phrenic nerve
d. .the oesophogeal aperture transmits the vagus nerves
e. .motor fibre supply is by the intercostals nerves

Concerning histological consequences.


a. .the round ligament contain smooth muscle fibres
b. .the bladder epithelium has no mucus glands
c. .the whole of the urethra is lined by squamous epithelium
d. .the sacroiliac joint has a synov ial membrane
e. .skene's gland are present in the clitoris

The pelvic splanchnic nerves.


a. .are derived from the posterior rami of the sacral spinal nerves
b. .supply afferent fibres
c. .mix with branches of the sympathetic pelvic plexus
d. .supply the ascending colon with motor fibres
e. .supply the uterus with parasympathetic fibres

The anal canal.


a. .has an upper part which is innervated by the inferior hypogastric plexus
b. .has a lower part which is supplied by the superior rectal artery
c. .drains lymph to the superficial inguinal nodes from its upper part
d. .has its internal sphincter innervated by the inferior rectal nerve
e. .has a superficial part of its external sphincter attached to the coccyx

The female urinary bladder.


a. .is in contact with the supravaginal uterine cervix
b. .is joined to the umbilicus by the urachus
c. .is separated from the posterior surface of the pubis by peritoneum
d. .is connected laterally to the tendinous arch of the pelvic fascia
e. .receives visceral afferent innervations from the pudendal nerve

1. The seminal vesicle.


a. .contain spermatids
b. .secrete hyaluronidase
c. .secrete acid phosphatase
d. .secrete fructose
e. .secrete prostaglandins
In the femoral triangle, the femoral artery is.
a. .crossed by the superficial circumflex iliac vein
b. .posterior to the femoral branch of the genitofemoral nerve
c. .medial to the long saphenous vein
d. .posterior to the femoral vein at the apex of the triangle
e. .medial to the femoral nerve

The external iliac artery.


a. .enters the thigh anterior to the inguinal ligament
b. .at its origin is crossed by the ureter
c. .at its origin is crossed by the ovarian vessels
d. .lies medial to the external iliac vein at its distal end
e. .gives rise to the deep external pudendal artery

Concerning the diaphragm.


a. .the vena caval opening lies within the central tendon
b. .it has an aperture for the oesophagus at the level of the 12 th thoracic vertebra
c. .it is attached to the pericardium
d. .motor innervations is received from the lower six intercostals nerves
e. .sensory innervations are derived solely from the anterior primary rami of the 3rd , 4th and 5th
cervical n

The obturator internus muscle.


a. .leaves the pelvis through the obturator foramen
b. .is inserted onto the greater trochanter of the femur
c. .forms the roof of the ischiorectal fossa
d. .is innervated by the femoral nerve
e. .has a fascia on its pelvic surface which gives origin to the levator ani
Concerning the rectus sheath.
a. .above the costal margin rectus abdominis lies on the costal cartilages
b. .below the arcuate line the internal oblique splits to enclose rectus abdominis
c. .it contains the musculophrenic artery
d. .it s innervated by the ilioinguinal nerve
e. .pyramidalis is innerv ated by the subcostal nerve

The ureter.
a. .is supplied in part by the ovarian artery
b. .lies lateral to the transverse processes of the lumbar vertebrae
c. .passes above the genitofemoral nerve
d. .is lined by a simple columnar epithelium
e. .passes below the uterine artery

Concerning the spinal cord and its meninges.


a. .the pain pathways run in the posterior columns
b. .the pathway for discriminative touch run in the posterior columns
c. .the cord in the adult terminates at the level of the third lumbar vertebra
d. .the dural sac (lumbar cistern)terminates at the level of the lumbosacral junction
e. .the anterior corticospinal tract carries uncrossed motor fibres

The following are sites of anastomosis between systemic and portal veins.
a. .lower third of the oesophagus
b. .sigmoid colon
c. .umbilicus
d. .terminal rectum
e. .ureters at the pelvic brim

The right ureter.


a. .is approximately 50 cm in length
b. .is partly covered by the duodenum
c. .crosses the genitofemoral nerve
d. .enters the bladder anteromedially
e. .receives part of its blood supply from the uterine artery

The adult pituitary gland.


a. .contains the paraventricular nuclei
b. .is related to the sphenoid bone
c. .lies inferiorly to the optic chiasma
d. .contains a pars tuberalis inferiorly
e. .is entirely ectodermal in origin

In the vulva.
a. .sebaceous glands are not present in the labia minora
b. .apocrine glands are present in the labia majora
c. .both surfaces of the hymen are covered by keratinized epithelium
d. .the Bartholin's glands are mucus secreting
e. .the clitoris is covered by columnar Intestinal Epithelium

Canal of nuck present in


A.Labia majora
B.Labia minora
C.Inguinal canal
D.Obturator fossa
E.Femoral canal

n the pelvis, the ureter.


a. .is supplied by the uterine artery
b. .is crossed inferiorly by he uterine artery
c. .lies more than 5 cm lateral to the supravaginal cervix
d. .is derived from the urogenital sinus
e. .has sensory fibres passing in the pelvic splanchnic nerves

False statement about anal triangle is


A.Medially levator ani and external anal sphincter
B.Fascia over obturator internus with Alcock's canal
C.Roof levator ani
D.Floor skin and fat
E.posteriorly urogenital perineum

The lower third of vagina has lymphatic drainage to which nodes?


A.Common illiac nodes
B.External illiac nodes
C.Internal illiac nodes
D.Superficial Inguinal nodes
E.Para aortic nodes

6.The inferior hypogastric plexus is an important plexus supplying the hind gut. What are its nerve
root origins?
L3–L5
L4–L5 and S1–S4
S2–S4
T8–T12 and L1–L3
T10–12 and L1–L2
Key is D
The answer is T10–12 and L1–L2. The inferior hypogastric plexus is a continuation of both hypogastric nerves. It is
derived from the lower three thoracic and upper two lumbar segments. The parasympathetic and sympathetic supply
of this plexus effects uterine contractions and vasoconstriction.

5.Which group of nerves all constitute branches of the pudendal nerve?


A.Internal pudendal nerve, perineal nerve, dorsal clitoral nerve
B.Mid rectal nerve, dostal clitoral nerve, perineal nerve
C.Perineal nerve and posterior clitoral nerve
D.Perineal nerve, dorsal clitoral nerve, inferior rectal nerve
E.Perineal nerve, inferior clitoral nerve, mid rectal nerve

4 A urinary catheter is vigorously inflated in the urethra at the time of caesarean section. Urine leaks
from the rupture into which anatomical space?
A.Into the anterior abdominal wall and mons pubis
B.Into the lesser pelvis
C.Into the peritoneal cavity
D.Into the vagina
E.Laterally to the femoral triangles

3.The inguinal canal transmits the round ligament and which important nerve?
A.Iliohypogastric nerve
B.Ilio inguinal nerve
C.Obturator nerve
D pelvic splanchic nerve
E.pudendal nerve

A 76 year old lady presents with painless Frank haematuria. A cysto scopy identifies a bladder
carcinoma .You organise a CT for Tumour for staging .To which lymph nodes are metastases most
likely?
A.External illiac
B.Deep inguinal
C.para aortic
D.pollutes
E.Superficial inguinal

Lymphatic drainage of the uterus


A.Fallopian tubes drain mainly to the para- aortic lymph nodes
B.Cervix is mainly to the external illiac nodes
C.Uterine cornue to the superficial inguinal lymph nodes
D.Body drains to mainly to the internal illiac lymph nodes
A.All of the above
Regarding Vagina which one is true
A.Has no somatic innervation
B.Has acidic OH ranges between 2-3
C.upper vagina is sensetive to pain
D.Musculosa is arranged in inner circular and outrer longitudinal fibers
E.Has a vertical opening slit

Regarding Perineal pouches


A.Ishiocavernosus is located in the deep pouch
B.Bartholin gland lies deep to the perineal muscle
C.External urether sphincter is located in the superficial pouch
D.Superficial transverse perineal muscle lies superficial to the dollar fascia
E.Deep layer of the colles fascia continue with the scarpas fascia in the abdomen

Ligaments of pelvic fascia


A.Ureter is related to the uterosacral ligament
B.Pubocervical ligament enclosed the uterine cervix
C.Uterine artery passes within the Mackenrodts ligament
D.Ureter lies underneath the cardinal the cardinal ligament through its course
E.Transverse cervical ligament maintains the anteversion position of the cervix

The vulva ( no key)


A.Clitoris is covered by simple cuboidal epithelium
B.Smooth muscle is only located in labia majora
C.Labia majora is innervated by nerves derived from the lumbar and sacral plexus
D.Lymphatic drainage is to the internal illiac nodes
E.Labia majora is covered by non keratinized startified squamous epithelium

External illiac artery


A.Terminates at the mid point of inguinal ligament by becoming femoral artery
B.Give rise to inferior epigastric artery
C.Give rise to deep external pudendal artery
D.Crosses the ovarian vessels at its origin
E.Crossed by genital branch of genito femoral nerve
The following muscles are inserted in the perineal body
A . internal anal spincters
B.Iliococcygeus
C.Ishiocavernosus
D.External uretheral sphibters
E.Coccygeus

The deep circumflex iliac artery is a branch of which artery?


A. Internal iliac artery
B. Internal pudendal artery
C. External iliac artery
D. Common iliac artery
E. Femoral artery

The following statements concerning the pelvic vessels are correct.


a. .the inferior gluteal artery passes through the greater sciatic foramen
b. .the inferior vesical artery supplies the fundus of the bladder
c. .the middle rectal artery supplies the rectal mucosa
d. .the uterine venous plexus communicates with the rectal plexus
e. .the vesical venous plexus drains into the external iliac artery

In the pituitary gland.


a. .the anterior lobe is smaller than the posterior lobe
b. .the posterior lobe is ectodermal in origin
c. .the acidophil cells produce oxytocin
d. .the basophil cells produce growth hormone
e. .the blood supply is derived from the internal carotid artery

In the vulva.
a. Blood is supplied by the internal pudendal artery.
b. The anterior parts of the labia majora are innervated by the obturator nerves.
c. The posterior parts of the labia majora are innervated by the genitofemoral nerve.
d. The lesser v estibular glands lie deep to the bulb of vestibule.
e. The round ligaments of the uterus terminate in the labia minora.

Fetal head diameters


A.submento- bregmatic is the presenting diameter in face presentation and measures 9.5 cm
B.Bregma reprents the root of the nose, which separates the face below from the brow above
C.Brow presentstion can not be delivered normally as the widest diameter will be the occipito frontal
diameter
D.Head circumference at term equals to 25cm
E.There are four fontanels in the fetal head at birth

the adult spinal cord.


a. .commences at the foramen rotandum
b. .ends at the level of upper lumbar vertebrae
c. .lies posterior to the spinal laminae
d. .is enclosed within a single layer of dura matter
e. .has a thoracic parasympathetic outflow
the vulva and perineum are supplied by the following nerves.
a. .sciatic
b. .posterior cutaneous nerve of the thigh
c. .inferior rectal
d. .ilioinguinal
e. .obturat

The pelvic splanchnic nerves.


a. .contain parasympathetic fibres
b. .unite with branches of the sympathetic pelvic plexus
c. .are preganglionic fibres
d. .supply the bladder sphincter with motor fibres
e. .supply the uterus with vasodilator fibres

In the abdominal wall.


a. .the rectus abdominis muscle is attached to the crest of the pubis
b. .te posterior border of the external oblique muscle ends in ht e linea semilunaris
c. .the aponeurosis of external oblique muscle takes part in the formation of the conjoint tendon
d. .the inferior epigastric artery is a branch of internal iliac artery
e. .the conjoint tendon blends medially with the anterior layer of rectus sheath

The obturator nerve.


a. Arises from segments L2,3,4.
b. Crosses the sacroiliac joint as it enters the pelvis.
c. Innervates the obturator internus muscle.
d. Is a lateral relation to the ovary.
e. Innervates the knee joint.

The pituitary gland.


a. Lies below the diaphragma sellae.
b. Is developed from 2 primordia.
c. Communicates with the hypothalamus.
d. Lies inferior to optic chiasma.
e. Is anterior to the sphenoidal sinus.

The left ureter in the female.


a. Develops from an outgrowth of the paramesonephric duct.
b. When radiologically visualized runs along the tips of the transverse processes of the lumbar
vertebrae.
c. Is narrowed in caliber as it crosses the pelvic brim. .
d. Runs medial to the ovary.
e. Lies anterior to the vagina as it enters the trigone of the bladder.

The adult female urethra.


a. Is 7 cm in length.
b. Is lined by columnar epithelium in its proximal half.
c. Has mucus gland in its distal third.
d. Passes through the perineal membrane.
e. Is surrounded by smooth muscle in the middle third.

Regarding the ovary, which of the following is correct?


A Disease (tumours) of the ovaries can cause referred pain to the umbilicus
B Lies in the ovarian fossa of the lateral pelvic wall behind the caecum
C Receives its principal arterial supply from the internal iliac artery
D The ovarian ligament is attached to the proximal third of the fallopian tube
E The ovarian vessels are contained within the mesovarium

Which of the following structures is attached to the perineal body?


A. Deep transverse perinei
B. internal anal sphincter
C. pubococcygeus
D. puborectalis
E. uterosacral ligament

What is the anatomical location of Bartholin’s gland?


A. Deep perineal pouch
B. Ischiorectal fossa
C. Pudendal canal
D. Superficial perineal pouch
E. Urogenital diaphragm

Which of the following leaves the pelvis via the greater sciatic foramen?
A.Pudendal Nerve
B.Dorsal Nerve to Clitoris
C .Perineal Nerve
D.Obturator Nerve
E.femoral nerve

The pudendal nerve travels through the greater sciatic foramen. What are its nerve root origins?
A.L5 & S1
B.S1 - S3
C.S4 & S5
D.S2 - S4
E none of the above

What is the boundary between the true and false pelvis?


Pelvic outlet
Pelvic Brim
Vagina
Sacrospinous ligament
Sacrotuberous ligament

The round ligament leaves the pelvis via what?


Greater sciatic notch
Lesser sciatic notch
Superficial inguinal ring
Deep inguinal ring
Lesser sciatic notch

An obstetrician performs a mediolateral episiotomy to expand the birth canal during a child birth.
Which of the following muscles is typically incised during this procedure?
A. Bulbospongiosus and superficial transverse perineal muscles
B. Bulbospongiosus and deep transverse perineal muscles
C. Bulbospongiosus and ischiocavernosu muscles
D. Ischiocavernosus and levator ani muscles
E. Bulbospongiosus and levator ani muscles

Which of the following statements about episiotomy is FALSE:


A. Median (midline) episiotomy is generally considered to be less painful the
mediolateral episiotomy.
B. Mediolateral or lateral episiotomy may be associated with more blood
loss than median one.
C. Indications for episiotomy include avoiding an imminent Perineal tear,
the use of forceps, breech delivery, & the delivery of premature infants.
D. The earlier the episiotomy is done during delivery, generally the more
beneficial it will be un speeding delivery.
E. Episiotomy incisions are repaired anatomically in layers.

Which paired structures contain uterine arteries and veins?


A- Broad ligaments
B- Cardinal ligaments
C- Pubocervical ligaments
D- Round ligaments
E- Uterosacral ligaments

Which vessels anastomose with the vaginal arteries to form the blood supply to the lower third of the
vagina?
A- Ovarian artery
B- Spiral arteries
C- Terminal branches of pudendal artery
D- Uterine artery
E- Vaginal branch of uterine artery

In the male urethra where do the ducts of the bulbourethral (Cowper's) glands enter?
Posterior wall prostatic urethra
lateral walls prostatic urethra
Membranous urethra

Lymph drainage from the bladder is via which nodes?


External, Internal and Common iliac nodes
Para-aortic nodes
Mesenteric nodes
Internal and Common iliac nodes only
Presacral

A baby with shoulder dystocia suffers a brachial plexus injury. The mother asks you if this will be
permanent. What percentage of babies will have permanent neurological dysfunction as a result of
brachial plexus injury secondary to shoulder dystocia?
<10%
15%
25%

The average functioning adult kidney contains approximately how many nephrons?
100
1,000
10,000
100,000
1,000,000

Bladder neck closure and relaxation of the bladder is mediated by?


Sympathetic Fibres L1,L2
Parasympathetic Fibres S2, S3,S4
Sympathetic Fibres S2,S3,S4
Parasympathetic Fibres T11,L1,L2
Sympathetic Fibres L3,L4,L5

A patient undergoes surgery for a vaginal vault prolapse. After surgery she complains of numbness
and groin pain. You assess her and find the numb area is the anterior aspect of the labia. Which
nerve has likely been damaged during surgery?
Ilioinguinal
Obturator
Femoral
Perineal
Genitofemoral

During the filling phase of micturition. At what bladder volume is the first urge to void felt?
15ml
50ml
150ml
300ml
500ml

The rectus sheath is formed by which of the following?


External oblique and rectus abdominus aponeuroses
Internal oblique and rectus abdominus aponeuroses
Rectus abdominus and pyramidalis aponeuroses
aponeuroses of internal and external oblique
aponeuroses of transversus abdominis, external and internal oblique

Where in the kidney is Renin secreted?


Podocytes
Distal convoluted tubule epithelial cells
Juxtaglomerular cells
Extraglomerular mesangial cells
Macula densa

The normal functional bladder capacity of a healthy human urinary bladder is approximately?
50ml
150ml
250ml
350ml
500ml

The posterior scrotal artery is a branch of which artery?


Internal Pudendal
External Pudendal
Inferior Rectal
Ilioinguinal
Superior rectal

The broad ligament contains which of the following structures?


Cardinal ligament
Ureters
Sacrospinous ligament
Uterine artery
Vaginal artery

Regarding lymph drainage of the fallopian tubes where does the majority of lymph drain to?
Lumbar nodes
Internal iliac nodes
External iliac nodes
Superficial inguinal nodes
Para-aortic nodes

The internal and external anal sphincters both receive innervation from which spinal segment?
S1
S2
S3
S4
S5

Regarding lymph drainage of the breast where does the majority of lymph drain to?
Axillary nodes
Parasternal nodes
Supraclavicular nodes
Interpectoral nodes

Regarding the uterine artery which of the following statements are FALSE?
Usually arises from the anterior branch of the Internal iliac artery
It crosses the Ureter posteriorly
It is the primary vessel responsible for arterial blood supply to the uterus
Typically anastomoses with branches of the Ovarian artery
Travels through the Broad Ligament of the Uterus

The male scrotum is typically how many degrees cooler than body temperature?
0.5 0C
1 oC
1.5 oC
2.0 oC
no difference

The juxtaglomerular apparatus (JGA) lies within which part of the kidney?
Renal Cortex
Renal Medulla
Minor Calyces
Major Calyces
Renal Pelvis

63.False statement among following:


A) Urinary bladder is lined by transitional epithelium
B) Parasympathetic supply via splanchnics nerve is motor to detrusor and inhibitory to sphincter
C) Urethra cotain inner longitudinal smooth muscle layer which shorten during micturition and outer
voluntary striated circular muscle which has a role in urinary incontinace
D) Breast is modified apocrine sweat gland
E) Growth of duct in breast is stimulated by progesterone

62.False statement regarding anal canal is


A) It is 4 cm long and external anal sphincter is striated one (extension of voluntary levator ani
muscle)
B) External anal sphincter is supplied by pudendal nerve
C) Internal anal sphincter is supplied by inferior hypogastric plexus and nervi erigentes
D) Below pectenline mucosa changes from columnar to squamous
E) Lymphatic drainage below intersphincteric groove is to rectum

61)False statement about rectum is:


A) 15 cm long
B) Acute angulation is produce by puborectalis
C) Has no taeniae or appendices epiploicae
D) Anterosuperiorly pouch of Douglas lies
E) Coloproctologist should be called if anorectal mucosal tear in vaginal delivery remain below
puborectalis

60)Wrong statement among following is:


A) Vagina is lined by keratinized stratified squamous epithelium
B) Labia majora contain hair follicle, sebaceous and apocrine gland
C) Vestibule describe as area in between clitoris anteriorly, labia minora anteriorly, fourchette
posteriorly and hymen superiorly
D) Greater vestibular gland of Bartholin`s open at 5 and 7 o`clock position posterolaterally at vaginal
introitus
E) Nerve supply of vulva is iliohypogastric,ilioinguinal, and pudendal nerve

59.True statement among following:


A) Uterus is maintain in anteverted position by transverse cervical ligament
B) Ureter lies above uterine vessels at level of isthmus
C) Patent processus vaginalis(canal of Nuck) leads to indirect inguinal hernia in female
D) Round ligament made up of smooth muscle fibres
E) Fallopian tube has inner longitudinal smooth muscle layer.

58.False statement regarding boundaries of ovarian fossa


A) Obturator vessels and nerves lies medially
B) Ampulla of uterine tube on medial surface
C) Bifurcation of common iliac artery above
D) Ureter and internal iliac artery and vein behind
E) Posterior border of ovary lies free of mesovarium

57.Pelvic peritoneum is supplied by:


A) Perineal branch of S4
B) Obturator nerve(L2-4)
C) Pudendal nerve
D) Nerve to obturator internus
E) Sciatic nerve

56.Pudendal nerve block is given by piercing needle just beyond ischial spine. Good test of efficacy
is
A) Loss of anal reflex
B) Relaxation of pelvic floor
C) Loss of sensation of vulva
D) Loss of sensation of lower third of vagina
E) All of the above

55.False statement among following about ischiorectal fossa is :


A) Posteriorly, the sacrotuberous ligament and gluteus maximus muscle
B) Medially fascia over levator ani and external anal sphincter
C) Laterally ischial tuberosity with obturator internus muscle
D) Pudendal (alcock) canal medially
E) Posteriorly, perineal branch of S4 and perforating cutaneous branch
54.Which of the following is not contain of superficial perineal pouch
A) Bulbospongiosus muscle
B) Ischiocavernosus muscle
C) Superficial transverse perineal muscle
D) External urethral sphincter
E) Corpus spongiosum

53.Pudendal nerve arises from


A) S2 to S4
B) S1 to S5
C) L4, L5,S1
D) L5 S1-2
E) S4

52.Which of the following is not branch of internal iliac artery:


A) Uterine artery
B) Ovarian artery
C) Inferior gluteal artery
D)Middle rectal artery
E) Internal pudendal artery

51.35 year old patient complain of pain in knee jont. USG 1 month back suggestive of ovarian cyst.
What can be cause of pain in knee joint:
A) Obturator nerve irritation
B) Lateral cutaneous nerve of thigh irritation
C) Femoral nerve irritation
D) Sacral nerve irritation
E) Genitofemoral nerve irritation

50.False statement among following is:


A) Right ovarian artery crosses inferior vena cava and is crossed by terminal ileal vein, caecal vein,
ileocolic vein
B) Right vein drain into right renal vein
C) Left common iliac artery is crossed by superior mesenteric vessels
D) Inferior epigastric artery and circumflex iliac artery arises just above inguinal ligament
E) Genitofemoral nerve lies anterior to psoas muscle

49).Right renal artery passes behind:


A) Diaphragm
B) Inferior vena cava
C) Aorta
D) Psoas muscle
E) Ureter

48.Which of the following is false related to acute appendicitis in preganancy:


A) Signs can be relatively subtle
B) Progression of pathology can be rapid due to failure of omentum to access problem and seal it off
C) Upward displacement of caecum
D) Surgical incision of appendicectomy at mc Burney`s point
E) Non of above

47.True statement among following is:


A) Superior pancreaticoduodenal artery is branch of superior mesenteric artery.
B) Inferior mesenteric artery is a branch of superior mesenteric artery
C) Superior mesenteric artery passes over superior to third part of duodenum
D) Inferior mesenteric artery terminate as middle rectal artery
E) Splenic artery give right gastro epiploic artery

46.Wrong statement regarding epiploic foramen is:


A) Superiorly caudate lobe of liver
B) Inferiorly first part of duodenum
C) Posteriorly inferior vena cava,
D) Anteriorly portal vein, hepatic artery, common bile duct
E) Left adrenal gland posteriorly

45.Wrong statement regarding femoral sheath is :


A) Femoral nerve lies in sheath
B) Femoral artery lies in sheath
C) Femoral vein lies in sheath
D) Femoral canal lies in sheath
E) Lacunar ligament lies medialy

44.Which of the following is not component of spermatic cord :


A) Vas deferens
B) Genital branch of genitofemoral nerve
C) Cremasteric artery
D) Inferior epigastric artery
E) Pampiniform venous plexous

43.Deep inguinal ring which is defect in fascia transversalis, is lies at:


A) Mid-inguinal canal point
B) Mid point of inguinal ligament
C) Divergent fibres of external oblique
D) Heselbach triangle
E) Femoral ring

42.Pyramidalis is supplied by:


A) Subcostal nerve
B) T8
C) T11
D) T10
E) T9

41).In order to avoid injury to inferior epigastric vessel needle for paracentesis must pass through:
A) Lateral to mc Burney`s point
B) Medial to mc Burney`s point
C) Medial to Palmer`s point
D) Lateral to Palmer`s point
E) Umbilicus

40.Bifurcation of abdominal aorta present at level of plane of iliac crests which is at level of :
A) T12
B) L3
C) L4
D) T1

39.Transpyloric level indicates:


A) Pylorus of stomach
B) Duodenojejunal flexure
C) Fundus of gall bladder
D) Renal hila and neck of pancreas
E) All of the above

38.Transpyloric plane is usually present hand breath below xiphod process , at ninth coastal
cartilage, marks termination of spinal cord is correspond to vertebral level:
A) T12
B) L1
C) T11
D) T10
E) T9

37.Aortic opening in the diaphragm usually present at the level of


A) T12
B) T8
C) T11
D) T10
E) T9

36.Inferior vena caval opening in the diaphragm usually present at the level of :
A) T12
B) T8
C) T11
D) T10
E) T9

35.Oesophageal opening in the diaphragm usually present at the level of :


A) T12
B) T8
C) T11
D) T10
34.Aspiration or insertion of chest drain is usually done through:
A) Mid clavicular line 5th intercostal space by running needle or drain over the rib
B) Mid axillary line 5th intercostal space by running needle or drain over the rib
C) Mid clavicular line 5th intercostal space by running needle or drain under the rib
D) Mid axillar line 5th intercostal space by running needle or drain under the rib
E) Mid clavicular line 4th intercostal space by running needle or drain over the rib

33.True ribs present:


A) 1 to 7
B) 8 to 10
C) 11 to 12
D) 1 to 8
E) 8 to 12

32.Angle of Louis lies at the level of :


A) T1
B) T2
C) T3
D) T4
E) T5

31.Presenting diameter of fetal skull is smallest with following presentation:


A) Occipital presentation
B) Face presentation
C) Vertex presentation
D) Brow presentation
E) Both a and b

30.Vitamin d deficiency cause:


A) Rachitic pelvis
B) Android pelvis
C) Anthropoid pelvis
D) Platypelloid pelvis
E) Gynaecoid pelvis

29. Obstetric conjugate is:


a) 10.5cm
b) 12cm
c) 13cm
d) 15cm
e) 17cm

28.Spread of malignancy from pelvic viscera to vertebral venous plexus system via:
a) Lateral sacral vessels
b) Lumbar vein
c) Posterior intercostal
d) Vertebral vein
e) Inferior mesenteric vein
27.IVC pierces diaphragm at level of :
a) L1
B) L2
C) T12
D) T1
E)T8

26.Common iliac vein join to form IVC behind right iliac vein at :
A) L1
B) L2
C) L3
D) L4
E) L5

25.External ilia artery become femoral artery beyond


A) L1
B) L2
C) L3
D) Inguinal ligament
E) Sacroiliac joint

24.Common iliac artery divides into internal and external iliac artery at
A) L1
B) L2
C) L3
D) L4
E) Sacroiliac joint

23.Terminal branches of aorta is


A) Right and left common iliac artery
B) Median sacral artery
C) Phrenic artery, suprarenal artery,
D) Renal artery and gonadal artery
E) Both a and b

22.Aorta arises divide into right and left common iliac artery at:
A) L1
B) L2
C) L3
D) L4
E) T12

21.Inferior mesenteric artery a ventral branch of aorta arises at


A) L1
B) L2
C) L3
D) L4
E)T10

20.Superior mesenteric artery a ventral branch of aorta arises at


A) L1
B) L2
C) L3
D) L4
E) T12

19.Coeliac artery a ventral branch of aorta arises at


A) L1
B) L2
C) L3
D) L4
E) T12

18.Aorta enters abdomen at:


A) L1
B) L2
C) L3
D) L4
E) T12

17.Obliterated remains of Urachus called:


A) Median umbilical ligament
B) Medial umbilical ligament
C) Ductus arteriosus
D) Ligamentum arteriosum
E) Ligamentum venosum

16.Ductus venosus become:


A) Medial umbilical ligament
B) Foramen ovale
C) Ductus arteriosus
D) Ligamentum arteriosum
E) Ligamentum venosum

15.Oxygenated blood from umbilical vein to inferior vena cava is bypasses by:
A) Ductus venosus
B) Foramen ovale
C) Dctus arteriosus
D) Ligamentum arteriosum
E) Ligamentum venosum

14)Cervix and upper one third of vagina drain into:


A) Superficial inguinal lymph node
B) Deep inguinal lymph node
C) External iliac nodes
D) Internal iliac nodes
E) Obturator nodes
13.Vulva drain into
A) Superficial inguinal lymph node
B) Deep inguinal lymph node
C) External iliac nodes
D) Internal iliac nodes
E) Deep femoral lymph node of cloquet

12.Clitoris drain into :


A) Superficial inguinal lymph node
B) Deep inguinal lymph node
C) External iliac nodes
D) Internal iliac nodes
E) Obturator nodes

11.Cisterna chyli lies over


A) L1,L2
B) T12,L1
C) L2,L3
D) L4,L5
E) T11,T12

In the male urethra which is the narrowest part


Prostatic urethra
Membranous urethra
Penile urethra
Spongy urethra
Cavernous urethra

A baby with shoulder dystocia suffers a brachial plexus injury. You diagnose Erb-Duchenne palsy.
Which nerve roots are typically affected?
C3 and C4
C4 and C5
C5 and C6
C6 and C7
C7 and T1

The external anal sphincter is innervated by which nerves?


Inferior rectal
Superior rectal
Pelvic Splanchnic
Ilioinguinal
genitofemoral

What testicular cell type secretes testosterone?


Myoid
Intersitial macrophages
Sertoli
Leydig
Germ
10.Acidophilic cells produce :
A) ACTH
B) FSH
C) TSH
D) GH
E) LH

9). Touch and vibration transmitted by


A) Lateral spinothalamic tract
B) Anterior spinothalamic tract
C) Posterior columns
D) Lateral spinocerebellar tract
E) Anterior cerebro spinal tract

8. Dura mater end at S2, epidural space is situated between:


A) Dura mater and spinal canal
B) Dura mater and arachnoid mater
C) Arachnoid mater and pia mater
D) Pia mater and spinal cord

7)Which of the following is parasympathetic effect:


A) Increase peristalsis
B) Constriction of coronary artery
C) Dilates bronchial tree
D) Relaxes detrusor muscle
E) Constrict smooth muscle sphincter

6. Which of the following is not sympathetic effect:


A) Iincrease heart rate
B) Constriction of coronary artery
C) Dilates bronchial tree
D) Relaxes detrusor muscle
E) Constrict smooth muscle sphincter

5. Preganglionic autonomic fibers are


A) Mylinated
B) Unmylinated
C) ACH is neurotransmitter at ganglion
D) Both a and c
E) Both band c

4. Somatic nervous system do not cross midline. Its posterior primary ramus supply:
A) Erector spinae and overlying skin
B) Cervical plexus
C) Brachial plexus
D) Lumber plexus
E) Sacral plexus
2. Vagina is lined by:
A) Simple squamous
B) Simple cuboidal
C) Simple columnar
D) Stratified squamous
E) Stratified cuboidal

1. Fallopian tube is lined by:


A) simple squamous
B) simple cuboidal
C) simple columnar
D) stratified squamous
E) stratified cuboidal

40.Puckering of breast tissue because of


a) Cowper`s ligament
b) Scarpa`s fascia
c) Fascia of camphor
d) Pectoralis major

39.Extravasated urine does not go into:


a) Superfascial perineal pouch
b) Thigh
c) Dartos fascia of scrotum
d) Scarpa`s fascia of anterior abdominal wall
e) None of above

38.Round ligament develops from:


a) Germinal ridge
b) Upper part of gubernaculum
c) Lower part of gubernaculum
d) Ectoderm
e) Sino vaginal bulb

37.Uterine muscle develops from:


a) Endoderm
b) Ectoderm
c) Mesoderm
d) Neural crest
e) All of above

36.Which of following structure not develop from paramesonephric duct:


a) Appendix of testes
b) Prostatic utricle
c) Duct of Gartner
d) Fallopian tube
e) Upper third of vagina
35.Which of following structure not develop from mesonephric duct:
a) Vas deference
b) Epoophoron and mesoophoron
c) Duct of gartner
d) Prostatic utricle
e) Epididymis

33.Branches of internal pudendal artery are:


a) Posterior labial branch
b) Artery to bulb of vestibule
c) Dorsal artery of clitoris
d) Deep artery of clitoris
e) All of above

31)Space between labia minora called:


a) Vulva
b) Vestibule
c) Perineum
d) Mons pubis
e) Bulb of vestibule

You are asked to assess a patients perineal tear following labour by vaginal delivery. You note a
laceration that extends through the external anal sphincter and has partially torn the internal anal
sphincter. How would you classify this tear?
1st
2nd
3b
3c

30.True statement among following is:


a) Mons pubis is made up of adipose tissue and cover with hair and sebaceous gland
b) Cleft between labia majora is termed as vestibule
c) Ischiocavernosus muscle cut during medio-lateral episiotomy
d) Clitoris contain pair of carpora cavernosa
e) Bulb of vestibule lies on each side of ischiocavernosus muscle

Which one of the structures is not CUT OR SPILT when making a Pfannestiel incision
A) scarpa s fascia
B) rectus Abdominus Muscle
C) linea Alba
D) transversalis Fascia
E) parietal peritoneum

29).Term vulva include:


a)Mons pubis
b)Labia majora and minora
c)Clitoris
d)Grater vestubular gland
e)All of above

28)False statement about ovary is:


a) Ovary is almond shape organ with diameter of 4 cm
b) External iliac vessel present in posterior relation of ovarian fossa
c) Ovarian fossa contain obturator nerve
d) Right ovarian vein drain into inferior vena cava
e) Left ovarian vein drain into left renal vein

Upper half of Anal canal is sensitive to which stimuli


A) stretch
B) stretch & touch
C) stretch , touch , and temperature
D) stretch , touch , temperature & pain
E) stretch & pain

4.. False statement about uterus is:


a) Anteriorly uterovesical pouch of peritoneum
b) Ureter is situated 12 mm lateral to lateral vaginal fornix
c) Peritoneum covers whole of uterus
d) Uterine artery is a branch of internal iliac artery and it is lies above ureter and supply uterus at
level of isthmus
e) Ovarian artery is a branch of aort

False statement about uterus is:


a) In nulliparous women external Os is circular
b) In pregnancy cervix has consistency like nose
c) Child cervix is twice size of body of uterus
d) Isthmus is 1.5cm in length

Regarding the ovarian artery which of the following statements is TRUE?


It arises from the Internal iliac artery
It arises from the External iliac artery
It arises from the Uterine artery
It arises from the Abdominal Aorta
It arises from the Renal artery

The arterial blood supply to the bladder is via branches of which artery?
Internal Iliac
External Iliac
Internal pudendal
Perineal artery
Inferior epigastric artery

A patient who is 36 weeks pregnant comes to see you as she has developed tingling to the
rightlateral thigh over the past 3 weeks. On examination there are no skin changes and no muscle
weakness. What is the likely diagnosis?
Obturator nerve entrapment
Pudendal nerve entrapment
Femoral Nerve entrapment
Meralgia Paraesthetica
Shingles

A patient undergoes oophorectomy and the ovarian veins are ligated. Which vein does the right
ovary drain into?
Right renal vein
Inferior vena cava
Right internal iliac
Right uterineAzygos

Regarding the Pituitary gland which of the following statements is true?


It rests on the Ethmoid bone in the skull base
Passes through the Foramen Rotundum
It is surrounded by the sella turcica
Is primarily an exocrine gland
Is part of the posterior cranial fossa

The femoral triangle is bounded medially by which of the following structures?


Inguinal ligament
Inguinal canal
Adductor longus
Adductor brevis
Sartorius

Regarding the pubic symphysis, what type of joint is it?


Fibrous
Condyloid
Synovial
Primary Cartilaginous
Secondary Cartilaginous

Which of the following is true regarding the femoral artery?


1. Is a branch of internal iliac
2. is crossed anteriorly by medical cutaneous nerve of thigh
3. lies lateral to iliopsoas
4. lies medial to the femoral vein
5. lies medial to the lymphatic vessels

What lies at the superior border of ovarian fossa...


A.external iliac artery & vein.
B.obturator nerve.
C.internal iliac artery &vein
D.sacral plexus
E.femoral n.
Pudendal nerve,enters the pelvis through-
a. greater sciatic foramen
b. lesser sciatic foramen
c. obturator foramen
d. lateral wall of ischiorectal fossa
e. sacral foramina

The anatomy of the spinal cord and dural space is important when giving regional spinal anesthesia.
At what approximate spinal level do the dural space and the spinal cord, respectively, end?
(A) T10, T8
(B) L2, T10
(C) L5, T12
(D) S2, L2
(E) S5, S2

Inhibin is secreted by..


A.sertoli cells
B.leyding cells
C.epithelial cells
Do.germ cells
Myoid cells

Proximal 2/3 of transverse colon is supplied by..


A.right colic a
B.Middle colic a
C.left colic a
D.Ascending branch of left colic a

The most common measured pelvimetry in clinics:


A - transverse diameter of inlet
B- pubic arch
C- iliac crest
D- laxity of perineum
E- AP diameter of outlet

Which one of the above statments about the anal sphincter is true?
A.the int. Anal sphincter is composed of skeletal ms
B. the int. Anal sphincter lies deep to the outer layer of smooth muscles
C. the ext. anal sphincter is composed of smooth muscles
D. the ext. anal sphincter is inserted into the coccyx
E. the ext. anal sphincter is inserted into the perineal body anteriorly

Insertion of intermediate fibers of levator ani.


A.Perineal body.
B.Annococygeal body.
C.Coccyx
D.Scarpa fascia
E.Fascia lata
A patient sustained a laceration of the perinium during delivery, it involved the
muscles of Perineal body but not the anal sphincter. Such a laceration would
be classified as :
A.First degree
B. Second degree
C. Third degree
D.Fourth degree

Which of the following structures separates the superficial perineal pouch from the deep perineal
pouch?
A. Superior levator ani
B. Inferior levator ani
C. Superior fascia of urogenital diaphragm
D. Perineal membrane
E. Colles fascia

superfial perineal pouch contains all but ...


A.Bulbospongious
B.Ischiocavernous
C. crura of clictoris
D.Bulbourethral glands

perineal body is point attachment of all


Except
A.external anal sphinecter
B.Bulbospongious
C.transvese perineal muscles
D.levator ani
E.ischorectal fossa

following forceps delivery a woman is noted to have a perineal injury involving the external and
internal anal sphincters as well as the anal epithelium, what degree of tear does she have?
A .2 degree
B. 3a
C .3b
D. 3c
E .4 degree

Sertoli cells contain receptors to which hormone?


Inhibin
Oestradiol
Testosterone
LH
FSH

A patient attends clinic with a vaginal prolapse. On examination the vagina protrudes by 1.5cm
below the plane of the hymen. According to the POPQ classification what grade is this prolapse?
Grade 0
Grade 1
Grade 2
Grade 3
Grade 4

Leydig cells contain receptors to which hormone?


LH
FSH
Testosterone
Oestradiol
Inhibin

Patient in labour with occipito posterior what is the presenting part?


Suboccipitofrontal
Suboccipitobregmatic
Occipitofrontal
Mentofrontal

Only structure that leaves through lesser schiatic foramen?


Tendon of obturator internes through lesser sciatic foramen

Most important ligament for support of urinary bladder ?


Pubocervical ligament

During a difficult delivery an obstetrician uses forceps to extract the infant.


Upon examining the baby you notice forceps impressions posteroinferior to the
ear. Which of the following is very likely to be damaged?
A. Trigeminal nerve
B. Facial nerve
C. Abducens nerve
D. Brachial plexus
E. C8 T1nerve

During childbirth, anesthesia is administered into the epidural space of the


spinal column. Where is the epidural space located?
A. Between supraspinous and interspinous ligaments
B. Between the wall of vertebral cavity and dura mater
C. Between arachnoid and dura mater
D. Between arachnoid and pia mater
E. Between pia mater and spinal canal

Which of the following nerves enters the thigh by passing beneath the
inguinal ligament, just medial to the anterior superior iliac spine?
A. Femoral nerve
B. Iliohypogastric nerve
C. Ilioinguinal nerve
D. Genitofemoral nerve
E. Lateral cutaneous nerve of thigh

Lymphatics of the sigmoid colon drain into


A. Superior mesenteric and Pre aortic lymph nodes
C. Inferior mesenteric and Pre aortic lymph nodes
B. Internal iliac lymph nodes
C. External iliac lymph nodes
E. Obturator lymph nodes

Nerve supply of pelvic part of Levator Ani muscle?


a. S1 S2
b. S2 S3
c. S3 S4
D.S4 S5

What is the nerve supply to the skin of the perineum?


A. Inferior gluteal
B. Inferior rectal
C. Deep perineal
D. Superficial perineal
E. Internal obturator

Which vessels pierce rectus abdominis....


A.superior epigastric vessels
B.inferior epigastric vessels
C.deep circumflex iliac c
Vessels
D.lateral thoracic vessels.

Lateral to femoral sheath is which nerve???


A.lateral cutaneous nerve
B.obturator nerve
C.illoinguinal nerve
D.genitofemoral nerve

Femoral sheath is derived from?


A.intra peritoneal extra abdominal fascia
B.extraperitoneal intra abdominal fascia
C.aponeurosis of external oblique
D.aponeurosis of fascia transversalis.

Lacunar ligament is .....


A.formed from the conjoint tendon.
B.part of posterior wall of rectus sheath.
c.not continuous with the inguinal ligament
D.an important medial relation to the femoral ring of the femoral sheath..
E. Attached to the interior ranks of pubis..
Branches of femoral artery include all of the follwing except:
A-superficial circumflex iluac artery.
B-deep external pudendal artery.
C-deep circumflex atery.
D-profunda femoris artery.

The inferior epigastric artery is a branch of which artery?


A External iliac artery
B Femoral artery
C Internal iliac artery
D Umbilical artery
E Uterine artery

The main blood supply of the vulva is:


A. Inferior hemorrhoidal artery.
B. Pudendal artery.
C. Ilioinguinal artery.
D. Femoral artery.
E. Inferior Hypogastric artery

The nerve supply to the vulva is derived from all of the following nerves, except:
A. The pudendal nerve
B. The ileo-inguinal nerve
C. The genito-femoral nerve
D. The posterior cutaneous nerve to the thigh
E. The inferior hemorrhoidal nerve

Nerve supplying posterior 2/3rd of vagina


A. Perineal nerve of S4
B. Posterior cutaneous branch of femoral nerve
C. Pudendal nerve
D.femoral nerve
A 36-year-old woman who had supraspinous ligament fixation 2 days ago
complains of pain over right mons pubis, right labia, and the perineum. Which
nerve is most likely to be injured?
A. Iliohypogastric nerve
B. Ilioinguinal nerve
C. Genitofemoral nerve
D. Pudendal nerve
E. Posterior femoral cutaneous nerve

The deep transverse perineal muscle


A.Originates from the perineal body
B.Is inserted onto the body of the pubic bone
C.Is supplied by the obturator nerve
D.Fixes the perineal body within the perineum
E.Lies within the superficial perineal pouch

This ligament forms two ridges on either side of the Pouch of Douglas
A.Round ligament
B.Infundibulo-pelvic ligament
C.Utero-sacral ligament
D.Ileo-pectineal ligament
E.Transverse cervical ligament

Regarding urogenital diaphragm:


A. Is located superior to pelvic diaphragm.
B. Is located inferior to pelvic diaphragm.
C. Anteriorly, it encloses the bladder.
D. Posteriorly, it encloses the superficial trasverse perineal muscle.
E. Support the pelvic organs.

all of the following pelvic structures support the vagina, EXCEPT


a. Perineal body
b. pelvic diaphragm
c. Levator ani muscle
d. Infudibulopelvic ligament

While doing vaginal hysterectomy which of the following nerves is prone to injury
A)sciatic
B)pudendal
C)common peroneal
D)obturator
E)lateral cutaneous nerve

Patient was diagnosed of having right ovarian teratoma of 6x8cm. RMI score is 25. Patient was
counselled for surgery and opted for laparoscopic cystectomy. What is the most common nerve
injury associated with this procedure?
a. sciatic nerve
b. femoral nerve
c. obturator nerve
d. genitofemoral nerve
e. ilioinguinal nerve

A 35-year-old diabetic woman underwent vaginal surgery. Postoperatively, she


has Loss of cutanous sensation over the anterior and lateral surface of the thigh.
The compression of which nerve is likely to be responsible?
A. Superior gluteal nerve
B. Lateral cutanous nerve
C. Sciatic nerve
D. Femoral nerve
E. Obturator nerve

The vulva & perineum are supplied by the following nerve:


A. Sciatic n
B, posterior cutaneous nerves of thigh
C. Superior rectal n
D. Illioinguinal n
E. Obturator n

A 32-year-old woman complains of pain in the right buttock. She is 36 weeks


pregnant and has a history of chronic back pain.
Which nerve supplies the gluteus maximus muscle?
A Inferior gluteal
B Internal obturator
C Internal obturator (lateral cutaneous nerve of the thigh)
D Sciatic
E Superior gluteal

Following radical hysterectomy and pelvic lymphadenectomy for Stage I


carcinoma of the cervix, 63-year-old woman reports numbness along the medial
thigh as well as weakness of hip adduction. Which nerve most likely to have been
injured during the operation?
A. Obturator nerve
B. Femoral nerve
C. Inferior gluteal nerve
D. Superior gluteal nerve
E. Sciatic nerve

Recall of March 16 & March 15


An 18-year-old woman who had inflammation of the right ovary complains painful spasms in the
muscles and some numbness on the skin of the medial part of her right thigh. The following nerves
is mostly involved?
A. Femoral nerve
B. Genitofemoral nerve
C. Sciatic nerve
D. Obturator nerve
E. Nerve to obturator internus
To reduce postoperative pain after caesarean section, an obstetrician performs
ilioinginal-iliohypogastric nerve block (IINB) on his 25-year-old patient. What is the
nerve root of ilioinguinal and iliohypogastric nerve?
A. T12
B. T12 and L1
C. L1
D. L2
E. L1 and L2

Motor fibers to detrusor muscle of the urinary bladder are derived from which
of the following nerves?
A. Pelvic splanchnic
B. Greater splanchnic
C. T11-L2
D. Superior hypogastric plexus
E. Sacral plexus

Blood supply of Bartholin gland?


A. Deep external pudendal artery
B. Internal pudendal artery
C. Superficial epigastric artery
D. Middle rectal artery

What is the blood supply of sigmoid colon?


A. Celiac trunk
B. Superior mesenteric artery
C. Inferior mesenteric artery
D. External iliac artery
E. Internal iliac artery

In what part of the female urogenital system describes the Vestibule?


A. Space between labium minus
B. Space between labium majus
C. Space between labia minora and majora
D. Area around hymen
E. Area between hymen and labia minora

Which of the following ligaments allows us to stand upright with a minimum of


muscular support?
A. Sacrospinous ligament
B. Sacrotuberous ligament
C. Iliolumbar ligament
D. Iliofemoral ligament
E. Pubofemoral ligament

A 30-year-old P1 + 0 woman presents with painful swelling in right posterolateral part of vagina and
fever for 3 days. Diagnosis of Bartholin’s abscess is made. Incision and drainage is planned. What is
the anatomical location of
Bartholin’s gland?
A. Deep perineal pouch
B. Ischiorectal fossa
C. Pudendal canal
D. Superficial perineal pouch
E. Urogenital diaphragm

The most important finding in an ideal obstetric pelvis at mid cavity level is ?
A. Shallow & straight side wall.
B. Sacrospinous ligament at least 3.5 cms.
C. No great projection of the ischial spines.
D. Ischeal bispinous diameter measuring 10 cms.
E. Smooth sacral curve.

Cephalopelvic disproportion in the absence of gross pelvic abnormality can be diagnosed by:
.A Ultrasound
B. A maternal stature of less than 158
C. Trial of labour
D. X-ray pelvimetry
E. Pelvic examination

The oblique diameter of pelvis is:


A. 11 cm
B. 12 cm
C. 13 cm
D. 10cm

right ureter course in the pelvis...


a. Crosses the external iliac artery at the sacroiliac joint
b. Crosses the common iliac artery at sacroiliac joint
c. Crossed by common iliac vessels at sacroiliac joint
d. Crossed by external iliac vessels at pelvic brim.
Type of pelvis with increased incidence of face to pubes is ???
A. Android
B.anthropoid
C.gynaecoid
D.platepelloid

Pelvis most likely to cause bladder fistula during labor is....


A. Gynaecoid
B.flat
C.android
D.anthropoid
The following are typical in the female bony pelvis EXCEPT:
A. Has a transverse diameter of the inlet greater than the antero-posterior
diameter.
B. Has an obstetric conjugate of 11-12 cm.
C. Is funnel shaped.
D. Has an obtuse greater sciatic notch.
E. Has a pubic angle greater than 90 degrees

All the following characteristics are applied to a pelvis favorable to vaginal


delivery EXCEPT:
A. Sacral promontory can not be felt.
B. Obstetric conjugate is less than 10 cm.
C. Ischial spines are not prominent.
D. Subpubic arch accepts 2 fingers.
E. Intertuberous diameter accepts 4 knuckles on pelvic exam

A pelvic inlet is felt to be contracted if :


A. The anterio-posterior diameter is only 12 cm.
B. The transverse diameter is only 10 cm .
C. Platypelloid pelvis.
D. The mother is short.
E. The patient had a previous C-section.

All true for The gynaecoid pelvis , except:---


A. is the commonest type of female pelvis
B. has a wider aneroposterior than transvers diameter at the prim
C. has a shallow , wide sacrosciatic notch
D. has parallel pelvic side – walls.
The most important finding in an ideal obstetric pelvis at mid- cavity level is:
A - shallow and straight side walls
B- No great projections of ischial spines
C- Ischial bispinous diameter measuring 10cms
D- smooth sacral curve
E- sacrospinous ligament at least 3. 5cms

All of the following structures are involved in the supportive role that the pelvic floor plays except the:
A. Lovator ani muscles
B. Uterus
C. Parametria
D. Bulbo-cavernous muscles

Which of the following muscles is NOT a constituent of the pelvic floor (diaphragm)?
A.iliococcygeal
B.Piriformis
C.Puborectalis
D.Pubococcygeus
E.Coccygeus

Forms the lateral wall of the ischio-rectal fossa


A. Skin
B. Anal canal
C. Obturator internus fascia
D. Vagina
E. Rectum

The most important muscle in the pelvic floor is:


A. Bulbo cavernousus.
B. Ischio-cavernosus.
C. Levator ani.
D. Superficial transverse Perineal muscle.
E. Deep transverse Perineal muscle.

which muscle forms the pelvic floor ?


A, levator ani and coccygeus
B , levator ani and piriformis
C, obturator internus and coccygeus
D, obturator internus and levator ani

In the normal human pelvis all of the following are true except:
A-the sacrum has five paired foramina anteriorly.
B-the obturator foramen is coverd completely by membrane.
C-the pubic bone contribute to the formation of the acetabular fossa.
D-the promontory of the sacrum is the upper anterior border of the first sacral vertebra.
Obturator nerve,leaves the pelvis through-
a. greater sciatic foramen
b. lesser sciatic foramen
c. obturator foramen
d. lateral wall of ischiorectal fossa
e. sacral foramina
Which is correct concerning cerebrospinal fluid :
A.Containing blood increases reabsorption
B.Is produced at approx 300 ml/hr by choroid plexus
C.Is reabsorbed by arachnoid granulations in sup sagital sinus.
D.Lies in subdural space
E.Procceds from lateral ventricle via foramen monro into 4 th ventricle

Regarding the Pituitary gland which of the following statements is true?


A.It rests on the Ethmoid bone in the skull base
B.Passes through the Foramen Rotundum
C.It is surrounded by the sella turcica
D.Is primarily an exocrine gland
E.Is part of the post cranial fossa

Which artery exits the pelvis via the greater sciatic foramen and re-enters via the lesser sciatic
foramen?
A.Superior Gluteal
B.Inferior Gluteal
C.Internal Iliac
D.Internal Pudendal
E.Superior Gluteal

The greater sciatic foramen transmit the following except:


(A) internal pudendal vessels.
(B) piriformis muscle.
(C) posterior femoral cutaneous nerve.
(D) sacrospinous ligament.
(E) sciatic nerve

When you hold the bony pelvis in the anatomical position, which of the following structures lie in the
same vertical plane?
A. Sacral promontory and pubic tubercles
B. Anterior superior iliac spines and the anterior aspect of the pubic symphysis
C. Posterior superior iliac spines and the posterior aspect of the ischial tuberosity
D. Ischial spines and the posterior border of the obturator foramen
E. Superior pubic rami and the greater sciatic notch

Obturator foramen is bounded by??


A.ilium & ischiun
B.sacrum & ischium
C.pubis & ischium
D.pubis & ilium
Which of the following is incorrect regarding the obturator nerve:
A. Emerge from the medial border of psoas
B. Is formed from the posterior division of the 2nd, 3d & 4th lumber nerves
C. Passes lateral to internal iliac vessels
D. Lies below the obturator foramen
E. Is separated from the normally sited ovary only by the pelvic peritoneum

Which of the following muscles leaves the lesser pelvis through the greater
sciatic foramen?
A. pubococcygeus
B. Iliococcygeus
C. Puborectalis
D. Piriformis
E. Coccygeus

What landmark marks where the external iliac artery becomes the femoral
artery?
A. Crest of the ilium
B. Inguinal ligament
C. Pubic tubercle
D. Symphysis pubis
E. Obturator foramen

Which of the following structure passes through the foramen of Morgagni (Larrey's triangle )??
A. Mesentery of the oesophagus
B. Azygous vein
C. Superior epigastric vessels
D. Left splanchnic nerve
E. IVC

The following structure take part in the formation of the anterior fontannelle in the fetal skull:
A. Frontal bone
B. Occipital suture
C. Sagital suture
D. Glabella
E. Frontal suture

Which of the following muscles is contained with superficial perineal pouch?


A.external urethral sphictrea.
B.deep transeverse perinius muscle.
C.compressed uretherae.
D.urethrovaginal sphicter
E. Ischiocavernosus muscle.

"it is directed downwards and medially. it also forms the lacunar ligament"
Which of the following describes the statement above?
a. pyramidalis muscle
b. internal oblique muscle
c. external oblique muscle
d. rectus abdominis muscle
e. transversus abdominis muscle

Which of the following leaves the pelvis via greater sciatic foramen ?
A.pudendal n.
B.dorsal n.of clitoris.
C.perineal n.
D.obturator n.
E. Femoral n.

Internal & external anal sphincter both recieves innervation from which spinal segment?
A.S1
B.S2
C.S3
D.S4
E.S5

Which structure blocks the blood supply to a loop of small intestine at the
femoral ring causing strangulated femoral hernia?
A. Inguinal ligament
B. Lacunar ligament
C. Pectineal ligament
D. Pubic tubercle
E. Pubis symphysis

Identify which of the following gives the correct pathway of piriformis muscle.
Origin Exits pelvis via Insertion
A. Ventral surface of sacrum Greater siactic notch Greater trochanter of femur
B. Ventral surface of sacrum Lesser siactic notch Greater trochanter of femur
C. Ventral surface of sacrum Greater siactic notch Lesser trochanter of femur
D. Dorsal surface of sacrum Greater siactic notch Greater trochanter of femur
E. Dorsal surface of sacrum Lesser siactic notch Lesser trochanter of femur

Which of the following describes the correct order of musculature of the anal
canal from deep to superficial?
1. Deep part of external sphincter
2. Subcutaneous part of external sphincter
3. Internal sphincter
4. Superficial part of external sphincter
A. 1,4,2,3
B. 2,1,4,3
C. 3,1,4,2
D. 3,2,1,4

An 18-year-old woman who had inflammation of the right ovary complains


painful spasms in the muscles and some numbness on the skin of the medial part
of her right thigh. The following nerves is mostly involved?
A. Femoral nerve
B. Genitofemoral nerve
C. Sciatic nerve
D. Obturator nerve
E. Nerve to obturator internus

A 36-year-old woman underwent a total abdominal hysterectomy for uterine


fibroids. Postoperatively she complains of loss of flexion of her left hip and
numbness over her left anterior and medial thigh. The compression of which nerve
is likely to be responsible?
A. Common peroneal nerve
B. Femoral nerve
C. Ilioinguinal nerve
D. Obturator nerve
E. Pudendal nerve

A woman undergoes an emergency caesarean section at full dilatation following


a failed trial of instrumental delivery. There is a lateral extension to the uterine
excision which is bleeding.
Identify which of the following gives the correct pairing of artery and its origin.
Artery Origin
A Internal pudendal Posterior division of internal iliac
B Ovarian artery Common Iliac
C Testicular artery Abdominal aorta
D Uterine artery Abdominal aorta
E Uterine artery Anterior division of the internal iliac

Which of the following does not insert into the perineal body?
A Bulbocavernosus
B External anal sphincter
C Ischiocavernosus
D Levator ani
E Transverse perineal

Superficial perineal pouch is bordered superiorly by..


Fascia of perinium.
B.gallaudt fascia.
C.deep perineal fascia.
D.perineal membrane
E. Bulbo spongiosus.

Lymph from scrotum drains to??


A lumbar nodes.
B.para aortic n.
C.hypogastric n.
D.deep inguinal n.
E. Superficial inguinal n.
Which of the following is the inferior border of deep perineal pouch..
A.perineal membrane.
B.superior fascia of urogenital diaphragm.
C.sphincter urethrae.
D.membranoous layer of superficial fascia.
E. Bulbo spongiosus

Regarding levator ani which statement is true?


A.arterial supply is from inf rectal a.
B.arterial supply is from superior rectal a.
C.Arterial supply is from internal iliac a.
D.arterial supply is from inf gluteal a.
E.arterial supply is from sup gluteal a

you are called to see a woman after a prolonged labor with instrumental delivery converted to
c.section.she complaints of pain n needles to left thigh.on exam ant left thigh including ant knee has
reduced sensation & there is reduced power in testing knee extension..wht is the likely diagnosis.
???
A.common peroneal palsy.
B.obturator palsy.
C.genitofemoral n palsy.
D.femoral n palsy.
E.left L2 nerve palsy.

Femoral triangle contains all of following structures except???


A.deep inguinal l nodes
B.sephano femoral junction
C.femoral sheath.
D.genital branch of Genito femoral nerve
E.femoral nerve

Which of the following arteries is a branch of the internal iliac artery?


A.Cystic a
B.Deep Circumflex a
C.Femoral a
D.Ovarian a
E.Uterine a

-Regarding placental anatomy??


A. The decidua capsularis forms part of the placenta.
B. The fetal side of the placenta is divided into 30-40 cotyledons.
C. The intervillous space contains fetal blood.
D. Anatomically the placenta id fully formed by 30 weeks.
E. Fetal blood vessels develop in the mesenchymal core of the chorionic villi.

8. What is the relationship of the pudendal nerve to the pudendal artery in the pudendal canal?
a. medial
b. superior
c. lateral
d. inferolateral
e. inferomedial

During hysterectomy ureter were demaged . using your knowledge of anatomy damage is at which
level ??
A. Infendibulopelvic ligament
B. Base of broad ligament
C. With uterine artery
D. With round ligament

What is the lateral boundary of pelvic outlet..?


A.pubic arch.
B.ischial tuberosity
C.pubic crest.
D.pubic tubercle.
A.pubic rami

What is the ant boundary of pelvic outlet..?


A.pubic arch.
B.ischial tuberosity
C.pubic crest.
D.pubic tubercle.
A.pubic rami

What's the most serious risk associated with insertion of veresse needle through the umbilicus in a
thin patient, an outcome which is unlikely with hassen technique...
A.bowel perforation
B.damage to liver.
C damage to ureter
D.inf epigastriv a injury
E.retroperitoneal vascular injury.
Which one of the above landmarks is correctly paired with its dermatome?
A Xiphoid process – T10
B Umbilicus – T12
C Simphysis pubis – L5
D Xiphoid process – T7
E Anterior superior iliac spine – S1

26 The nerve supply to the rectus abdominis muscle?


A Upper 6 thoracic nerves
B Lower 6 thoracic nerves
C Upper 3 lumbar nerves
D Lower 3 lumbar nerves
E 12th thoracic nerve

With respect to hernias of the anterior abdominal wall


A Direct inguinal hernias are more common than
indirect hernias
B The sac of indirect inguinal hernias lies below the
pubic tubercle
C The sac of direct inguinal hernias lies below the
pubic tubercle
D The sac of direct inguinal hernias lies medial to
the pubic tubercle
E The sac of indirect inguinal hernias lies lateral to
the pubic tubercle

The sac of this hernia lies below and lateral to the pubic tubercle
A Direct inguinal hernia
B Indirect inguinal hernia
C Femoral hernia
D Richcter’s hernia
E Epigastric hernia

Which one of the above most accurately describes the Dentate line?
A Mucosal fold overlying inferior rectal artery
B Mucosal fold over anal columns
C Boundary between ectodermal and mesodermal
origin of anal canal
D Boundary between ectodermal and endodermal
origin of anal canal
E Boundary between mid-gut and hind-gut origins of
the anal canal

Which one of the above organs is correctly paired with its parasympathetic nerve supply?
A Urinary bladder – lumbar splanchnic nerves
B Small intestine – pelvic splanchnic nerves
C Descending colon – vagus nerve
D Vagina – lumbar splanchnic nerves
E Ovary – vagus nerve

The origin of the piriformis muscle


A Greater trochanter
B Lesser trochanter
C Body of sacral vertebrae
D Lateral mass of sacral vertebrae
E Simplysis pubis

Sensory innervation of the vagina is provided by which nerve?


A Dorsal nerve of the clitoris
B Inferior hypogastric plexus
C Inferior rectal nerve
D Obturator nerve
E Pudendal nerve
Which artery supplies the structures derived from the foregut of the embryo?
A Coeliac trunk
B Inferior mesenteric
C Middle rectal
D Renal
E Superior mesenteric

Which of the following arteries is a terminal branch (not paired) of the abdominal
aorta?
A Gonadal
B Median sacral
C Phrenic
D Renal
E Suprarenal

Following a routine elective caesarean section, the rectus sheath is being sutured.
With regards to the rectus sheath which of the following is correct?
A Arcuate line demarcates the upper limit of the posterior layer of rectus
sheath
B External oblique aponeurosis forms the posterior aspect of the sheath
C Internal oblique aponeurosis always passes in front of rectus abdominis
D Scarpa’s fascia is superficial to Camper’s fascia and the external oblique
E Transversalis fascia lies directly below the rectus sheath

Which of the following does not form a boundary of the femoral triangle?
A Adductor longus
B Inguinal ligament
C Obturator internus
D Pectineus
E Sartorius

A 21-year-old woman undergoes a laparoscopic ovarian cystectomy to remove


a dermoid cyst. Three days after the operation, she presents to the emergency
department feeling unwell and her haemoglobin level is found to be 6 g/dL.
Damage to a blood vessel is suspected from the laparoscopic procedure.
Which vessel crosses the common and external iliac artery in the infundibulopelvic
fold?
A Femoral artery
B Inferior mesenteric artery
C Median sacral artery
D Ovarian artery
E Renal artery

What is the nerve root of the ilioinguinal nerve?


A T12 and L1
B L1
C L1 and L2
D L2
E L2 and L3
Which nerve arises from ventral rami of lumbar plexus and emerges from the
medial border of psoas major muscle?
A. Femoral nerve
B. Genitofemoral nerve
C. lateral femoral cutaneous nerve
D. Common peroneal nerve
E. Obturator nerve

The anatomical structure to which the levator ani muscle is attached is:
a. cervix
b. syphysis pubis
c. obturator internus fascia
d. obturator foramen
e. lateral pelvic wall

Which of the following nerve passes through the rectus sheath?


a. iliohypogastric nerve
b. ilioinguinal nerve
c. subcostal nerve
d. lateral cutaneous nerve
e. femoral nerve

Which ascending artery can be damaged during open appendicectomy?


a. iliolumbar artery
b. superficial circumflex artery
c. deep circumflex artery
d. superior gluteal artery
e. inferior gluteal artery

Linea semilunaris is the lateral border of which muscle?


a. external oblique
b. internal oblique
c. rectus abdominis
d. pyramidalis
e. transversus abdominis

1. A patient was subjected to an emergency lower segment cesarean section for obstructed labour.
The anesthesiologist palpated the iliac crest to locate the site for spinal anesthesia. What is the
vertebral level of iliac crest?
a. L1
b. L2
c. L3
d. L4
e. L5

while performing laparoscopy, the lateral umbillical folds can be seen on the anterior abdominal wall.
this structure contains...
a. inferior epigastric vessels
b. obturator vessels
c. remnant of urachus
d. obliterated umbillical arteries
e. ovarian vessels

With regard to anal canal which one is true?


A. The upper half is line with cuboidal epithelium
B. The lower half is lined by keratinised stratified squamous epithelium
C. The fibers of ischiococcygeus form part of the internal anal sphincter
D. The dentate line lies at the border of the upper one third and lower two thirds of the anal canal
E. Hilton's line indicates the junction between keratinised and non-keratinised stratified squamous epithelium

During c section,rectus sheath is divided to reveal rectus muscle.the combined aponeurosis of which
muscles form the anterior component of rectus sheath??
A.external & internal oblique
B.external oblique & t.abdominis
C.internal oblique & t.abdominis
D.pyramidalis & serratus anterior
E.pyramidalis & t.abdominis

Arcuate line forms the border of pelvic brim,where it is located??


A.ilium
B.ischium
C.pubis.
D.sacrum
E.coccyx

A woman presents with advanced cervical cancer presents to hosp with worsening left loin pain
.which structure is most likely have become obstructed ???
A.cervix.
B.ureter
C.urethra
D.uterine artery
E.vagina

Motor supply to levator ani is derived mainly from....


A.L5
B.S1
C.S2
D.S3
E.S4

4. Which structure is attached to anterior superior iliac spine?


a. inguinal nerve
b. pubococygeus
c. inguinal ligament
d. psoas major
e. obturator fascia

Which of the following is a branch of pudendal nerve?


a. superior rectal nerve
b. middle rectal nerve
c. inferior rectal nerve
d. obturator nerve
e. pelvic splanchnic nerve

which of the following takes part in the arterial supply of ovary??


a.uterine a.
B.vaginal a
C.renal a
D.ureteric a.
E.none of above

Sacro iliac joint is which type of joint??


A.codyloid
B.synovial
C.primary cartilaginous
D.secondary cartilaginous
E.fibrous..

The arterial supply of bladder is via branches of which artery??


A.internal iliac artery.
B.external iliac artery
C.internal pudendal.
D.perineal a.
E.inf epigastric a.

Parasympathetic supply of bladder is from..


A.T10-L2
B.s1-s2
C.S2 -S4
D.s3 -s4
E. L5-S1

.The inferior hypogastric plexus is an important plexus supplying the hind gut. What are its nerve
root origins?
a.L3–L5
b.L4–L5 and S1–S4
c.S2–S4
d.T10–12 and L1–L2
e.T8–T12 and L1–L3

Which of the following nerve is transmitted through superficial inguinal ring?


A.femoral n.
B.genito femoral n.
C.ilio inguinal n.
D.peroneal n.
E.sciatic n.

somatic nerve supply of vagina is


A-superior hypogastric plexus
B-pelvic Splanchnic NERVE
C-pudendal Nerve

The arterial supply of bladder is via branches of which artery??


A.internal iliac artery.
B.external iliac artery
C.internal pudendal.
D.perineal a.
E.inf epigastric a.

11. What is the fetal diameter of mentovertical in brow presentation?


a. 9.5cm
b. 10cm
c. 10.5cm
d. 12.5cm
e. 13cm

5. The aorta bifurcates at which vertebral level?


a. L1
b. L2
c. L3
d. L4
e. L5

WHICH VESSELS CAN BE POSSIBLY INJURED IN THE SUBCUTANEOUS TISSUE WHEN A


TRANSVERSE SUPRAPUBIC SKIN INCISION IS MADE?
A. SUPERFICIAL EPIGASTRIC
B. SUPERFICIAL CIRCUMFLEX ILLIAC
C. DEEP CIRCUMFLEX ILLIAC
D. SUPERIOR EPIGASTRIC
E. INFERIOR EPIGASTRIC
while performing laparoscopy, the lateral umbillical folds can be seen on the anterior abdominal wall.
this structure contains>
a. inferior epigastric vessels
b. obturator vessels
c. remnant of urachus
d. obliterated umbillical arteries
e. ovarian vessels

Which of the following structures is the origin of the uterus?


A) Genital tubercle
B)Mesonephric duct
C )Metanephric blastema
D)Paramesonephric duct

Which of the following nerves supplies the gastrocnemius muscle?


A Axillary nerve
B Median nerve
C Radial nerve
D Tibial nerve
E Ulnar nerve

A 71‐year‐old woman presents to the orthopaedic outpatient clinic with left‐sided foot drop and altered
sensation below the lateral side of the knee. There is loss of knee flexion and absent ankle jerk.
She had undergone left total hip replacement six weeks ago.Which of the following nerves is most likely
to be damaged in this scenario? A Femoral branch of the genitofemoral nerve B Femoral nerve C Lateral
cutaneous nerve of thigh D Lateral plantar nerve E Sciatic nerve

Which of the following vessels supplies the medial and superior surfaces of the brain?
A Anterior cerebral artery
B Anterior communicating arteries
C Middle cerebral artery
D Posterior cerebral artery
E Posterior communicating arteries

Which of the following dermatomes supplies the nipple?


A C7
B L5
C S1
D T4
E T10

In relation to lymphatic drainage of the breasts, which one of the following options best describes the
site from where lymph drainage arises?

A Infraclavicular lymph nodes


B Lateral group of axillary lymph nodes

C Parasternal lymph nodes

D Pectoral group of axillary lymph nodes

E Subareolar plexus

Which one of the following vessels supplies the lateral surface of the brain?
A Anterior cerebral artery
B Anterior communicating arteries
C Middle cerebral artery
D Posterior cerebral artery
E Posterior communicating arteries

Which of the following dermatomes supplies the middle finger? A C7 B L5 C S1 D T4 E T10

Which of the following nerves supplies the opponens pollicis muscle? A Axillary nerve B Median nerve C
Radial nerve D Tibial nerve E Ulnar nerve

Which one of the following vessels supplies the inferior surface of the brain and the occipital lobe? A
Anterior cerebral artery B Anterior communicating arteries C Middle cerebral artery D Posterior cerebral
artery E Posterior communicating arteries

Which of the following nerves supplies the deltoid muscle? A Axillary nerve B Median nerve C Radial
nerve D Tibial nerve E Ulnar nerve

26‐year‐old female presents with right iliac fossa pain and is taken to theatre for an appendicectomy.An
incision is made through the skin and onto muscle with fibres passing superiorly in an oblique
direction.Which abdominal wall muscle does this describe?
A.Cremaster B.External oblique C.Internal oblique D.Pyramidalis E.Rectus abdominis

Whilst examining the abdomen of a 21‐year‐old female with abdominal pain you notice a well‐ defined
'six‐pack'.
Which abdominal wall muscle does this describe? A Cremaster B External oblique C Internal oblique D
Pyramidalis E Rectus abdominis

In relation to lymphatic drainage of the breasts, to which of the following sites does the majority of the
lymph initially drain? A Central group of axillary lymph nodes B Infraclavicular lymph nodes C Lateral
group of axillary lymph nodes D Parasternal lymph nodes E Pectoral group of axillary lymph

Which of the following is true of the common bile duct? A Lies posterior to the portal vein BLies
posterior to the second part of the duodenum C Lies to the left of the gastroduodenal artery D Lies to the
right of the hepatic artery E Opens into the third part of the duodenum

Potrebbero piacerti anche